JV’s Jatin Verma’s IAS Academy

JV’s Scholarship Mock Test

Jv’s Scholarship Mock Test helded on

5th September,2020.

[Time Allotted: 2 Hours] [Maximum Marks: 200] No. of Questions = 100

GENERAL INSTRUCTIONS 1. IMMEDIATELY AFTER THE COMMENCEMENT OF THE EXAMINATION, YOU SHOULD CHECK THAT THIS BOOKLET DOES NOT HAVE ANY UNPRINTED, TORN OR MISSING PAGES ETC. IF SO, GET IT REPLACED BY A NEW TEST BOOKLET. 2. You have to enter your Roll Number on the Test Booklet in the Box provided alongside. Do NOT write anything else on the Test Booklet. 3. This Test Booklet contains 100 Questions. Each question is printed in English. Each question comprises four responses (answers). You are to select the correct response which you want to mark on the Answer Sheet. 4. All questions carry equal marks. Attempt all questions. Your total marks will depend only on the number of correct responses marked by you in the answer sheet. For every incorrect response 1/3rd of the allotted marks will be deducted. 5. If a candidate marks more than one option in any answer, it will be treated as a wrong answer even if one of the marked options happens to be the right answer. 6. You have to mark all your responses ONLY on the separate Answer Sheet provided. See direction on the answers sheet. 7. Before you proceed to mark the response in the Answer sheet to various questions, you should fill in some particulars in the answer sheets as per the instructions. 8. After you have completed filling in all responses on the answer sheet and the examination is concluded, you should hand over to Invigilator only the answer sheet. You are permitted to take away with you the Test Booklet. 9. Mobile/Tablet/Bluetooth/Pager or any communication device is not allowed in the examination hall. 10. Use only BLACK BALL POINT PEN. 11. You are not allowed to leave the examination hall before the test duration is over. 12. Sheet for rough work are appended in the Test Booklet at the end.

DO NOT OPEN THIS BOOKLET UNTIL YOU ARE ASKED TO DO SO

E mail: [email protected] 8882932364 Answerkey

Q.1) ‘Gupkar Declaration’ is sometimes mentioned in the news with reference to Ans) b a) Eradication of child labour from by Exp) Parliament’s Power to Reorganise the 2025. States b) Special status of Jammu and Kashmir. Article 3 authorises the Parliament to: c) Increasing the participation of women in (a) form a new state by separation of territory employment. from any state or by uniting two or more states or d) Better relations between India and Pakistan. parts of states or by uniting any territory to a part of any state, Ans) b (b) increase the area of any state, Exp) Last year, the Gupkar Declaration was (c) diminish the area of any state, signed in a meeting of the mainstream political (d) alter the boundaries of any state, and leadership of Kashmir, during which they (e) alter the name of any state. unanimously resolved that “all the parties would be united in their resolve to protect and defend However, Article 3 lays down two conditions in identity, autonomy and special status of the JK this regard: one, a bill contemplating the above against all attacks and onslaughts changes can be introduced in the Parliament only whatsoever.” The meeting was convened at with the prior recommendation of the Srinagar MP Farooq Abdullah’s residence on President; and two, before recommending the Gupkar Road, which is how the name of the bill, the President has to refer the same to the document came to be known. state legislature concerned for expressing its views within a specified period. Source) https://www.thehindu.com/news/national/other- The President (or Parliament) is not bound by the states/six-jk-parties-forge-alliance-to-fight-for- views of the state legislature and may either pre-august-5- accept or reject them, even if the views are position/article32419437.ece?homepage=true & received in time. Further, it is not necessary to https://indianexpress.com/article/india/gupkar- make a fresh reference to the state legislature declaration-jammu-and-kashmir-abrogation- every time an amendment to the bill is moved and special-status-6565380/ accepted in Parliament. In case of a union territory, no reference need be made to the Q.2) Consider the following statements: concerned legislature to ascertain its views and 1. A bill for creation of a new state can only be the Parliament can itself take any action as it introduced in the House of the People. deems fit. 2. Indian territory can be ceded to a foreign state only by amending the Constitution under Moreover, the Constitution (Article 4) itself Article 368. declares that laws made for admission or Which of the statements given above is/are establishment of new states (under Article 2) and correct? formation of new states and alteration of areas, a) 1 only boundaries or names of existing states (under b) 2 only Articles 3) are not to be considered as c) Both 1 and 2 amendments of the Constitution under Article d) Neither 1 nor 2

368. This means that such laws can be passed by a) 1 only a simple majority and by the ordinary b) 2 only legislative process. Thus, a bill for creation of a c) Both 1 and 2 new state can be introduced in either House of the d) Neither 1 nor 2 Parliament. Hence, statement 1 is incorrect. Ans) d Does the power of Parliament to diminish the Exp) Inter-State Trade and Commerce areas of a state (under Article 3) include also the Articles 301 to 307 in Part XIII of the power to cede Indian territory to a foreign Constitution deal with the trade, commerce and country? This question came up for examination intercourse within the territory of India. before the Supreme Court in a reference made by the President in 1960. The decision of the Central Article 301 declares that trade, commerce and government to cede part of a territory known as intercourse throughout the territory of India shall Berubari Union (west Bengal) to Pakistan led to be free. The object of this provision is to break political agitation and controversy and thereby down the border barriers between the states and necessitated the Presidential reference. The to create one unit with a view to encourage the Supreme Court held that the power of Parliament free flow of trade, commerce and intercourse in to diminish the area of a state (under Article 3) the country. The freedom under this provision is does not cover cession of Indian territory to a not confined to inter-state trade, commerce and foreign country. Hence, Indian territory can be intercourse but also extends to intra-state trade, ceded to a foreign state only by amending the commerce and intercourse. Thus, Article 301 will Constitution under Article 368. Consequently, be violated whether restrictions are imposed at the 9th Constitutional Amendment Act (1960) the frontier of any state or at any prior or was enacted to transfer the said territory to subsequent stage. Pakistan. Hence, statement 2 is correct. The freedom guaranteed by Article 301 is a Source) freedom from all restrictions, except those which https://www.thehindu.com/news/national/other- are provided for in the other provisions (Articles states/six-jk-parties-forge-alliance-to-fight-for- 302 to 305) of Part XIII of the Constitution itself. pre-august-5- These are explained below: position/article32419437.ece?homepage=true & Indian Polity by M Laxmikanth, Chapter – 5 (i) Parliament can impose restrictions on the ‘Union and its Territory’. freedom of trade, commerce and intercourse between the states or within a state in public Q.3) Consider the following statements: interest. But, the Parliament cannot give 1. The legislature of a state cannot impose preference to one state over another or restrictions on the freedom of trade, discriminate between the states except in the case commerce and intercourse with other states. of scarcity of goods in any part of India. 2. The legislature of a state cannot impose any tax on goods imported from other states. (ii) The legislature of a state can impose Which of the statements given above is/are reasonable restrictions on the freedom of correct? trade, commerce and intercourse with that

state or within that state in public interest. But, 4. Act as a nodal agency for achieving the goals a bill for this purpose can be introduced in the of Universalisation of Elementary Education legislature only with the previous sanction of the President. Further, the state legislature cannot Select the correct answer using the code given give preference to one state over another or below. discriminate between the states. (Statement 1 is a) 1 and 2 only incorrect.) b) 2, 3 and 4 only c) 1, 2 and 3 only (iii) The legislature of a state can impose on d) 1, 2, 3 and 4 goods imported from other states or the union territories any tax to which similar goods Ans) d manufactured in that state are subject. This Exp) National Council of Educational provision prohibits the imposition of Research and Training (NCERT) discriminatory taxes by the state. (Statement 2 is The National Council of Educational Research incorrect.) and Training (NCERT) is an autonomous organisation set up in 1961 by the Government (iv) The freedom (under Article 301) is subject to of India to assist and advise the Central and State the nationalisation laws (i.e., laws providing for Governments on policies and programmes for monopolies in favour of the Centre or the states). qualitative improvement in school education. The Thus, the Parliament or the state legislature can major objectives of NCERT and its constituent make laws for the carrying on by the respective units are to: undertake, promote and coordinate government of any trade, business, industry or research in areas related to school education; service, whether to the exclusion, complete or prepare and publish model textbooks, partial, of citizens or otherwise. supplementary material, newsletters, journals and develops educational kits, multimedia digital Source) materials, etc. organise pre-service and in-service https://www.thehindu.com/news/national/corona training of teachers; develop and disseminate virus-unlock-3-centre-asks-states-not-to-put- innovative educational techniques and restrictions-on-inter-state-movement-of-people- practices; collaborate and network with state goods/article32419488.ece?homepage=true & educational departments, universities, NGOs and Indian Polity – M Laxmikanth, Chapter – 15 other educational institutions; act as a clearing ‘Inter-State Relations’. house for ideas and information in matters related to school education; and act as a nodal Q.4) Which of the following is/are the objectives agency for achieving the goals of of the National Council of Educational Research Universalisation of Elementary Education. In and Training (NCERT)? addition to research, development, training, 1. Prepare and publish model textbooks extension, publication and dissemination 2. Develop and disseminate innovative activities, NCERT is an implementation educational techniques and practices agency for bilateral cultural exchange 3. Act as a clearing house for ideas and programmes with other countries in the field of information in matters related to school school education. The NCERT also interacts and education works in collaboration with the international organisations, visiting foreign delegations and

offers various training facilities to educational reconstituted in 1989, 1993, 1997, 2002 and personnel from developing countries. 2015. NCDS acts as a forum for exchange of views on various issues pertaining to dam safety. Hence, all the options given above are correct. It is not a constitutional body. Hence, statement 1 is incorrect. Source) https://indianexpress.com/article/education/27- The National Committee, in consultation with the central-school-kids-have-no-phone-laptop-to- concerned State Government and the Ministry of access-classes-study-6561856/ & Water Resources, may further set up sub- https://ncert.nic.in/about-us.php?ln= committees to monitor the safety aspect of selected inter-state dams. The sub-committee Q.5) With reference to the National Committee submits its report to the National Committee who on Dam Safety (NCDS), consider the following after deliberations in NCDS will forward it to the statements: Ministry of Water Resources for appropriate 1. It is a constitutional body on dam safety decision and communication to the concerned established under Article 262 of the State Governments for the necessary follow up Constitution. action. 2. Union Jal Shakti Minister is the ex officio chairperson of this Committee. The National Committee on Dam Safety (NCDS) Which of the statements given above is/are is headed by the Chairperson of the Central correct? Water Commission. Hence, statement 2 is a) 1 only incorrect. b) 2 only c) Both 1 and 2 Functions: d) Neither 1 nor 2 · NCDS works for devising Dam Safety Policies and regulations for maintaining best Ans) d practices and standards of dam safety so as to Exp) National Committee on Dam Safety prevent any dam safety related disasters. (NCDS) · NCDS acts as a forum for analysing and The safety practices presently vary from State to discussing distress conditions in specified State. In order to evolve a uniform simplified dams and appurtenant structures. Such procedure based on the latest 'State-of-the Art' exchange of views will evolve best techniques, the Government of India techniques to be adopted for tackling the constituted a Standing Committee in August problem. 1982 to review the existing practices of · The committee may evolve and suggest inspection/maintenance of dams and allied changes in the Planning, Specifications, structures in various States and to evolve standard Construction and Operation & Maintenance guidelines for the same. The Standing Committee practices by analysing the causes of major was reconstituted with wider representation and historical dam incidents and dam failures. focused and pin pointed mandate in October 1987 · It acts as an advisory body to the Central in the name of National Committee on Dam Government on specific matters related to the Safety (NCDS). Thereafter, the NCDS has been dam safety as referred to it and also makes

recommendations to the Government in ernment-launches-new-jal-shakti-ministry- respect of dams located outside the territory 64869 of India. · It may evolve methods and make Q.7) Which of the following activities is/are recommendations for rehabilitation of old carried out by the Bureau of Indian Standards dams. (BIS)? 1. Setting standard for drinking water Source) 2. Setting standard for imported toys https://indianexpress.com/article/cities/hyderaba 3. Hallmarking of gold jewellery d/telangana-powerhouse-fire-srisailam-dam- Which of the statements given above is/are death-toll-6564812/ & correct? http://cwc.gov.in/damsafety/NCDS a) 1 and 2 only b) 2 only Q.6) Consider the following: c) 1 and 3 only 1. Ministry of Water Resources, River d) 1, 2 and 3 Development and Ganga Rejuvenation 2. Ministry of Drinking Water and Sanitation. Ans) d 3. Ministry of Shipping Exp) Bureau of Indian Standards (BIS) - The Bureau of Indian Standards is the National Which of the above Ministries has been merged to form the Ministry of Jal Shakti? Standard Body of India established under the BIS Act 2016 for the harmonious development of a) 1 only the activities of standardization, marking and b) 1 and 2 only quality certification of goods and for matters c) 2 and 3 only connected therewith or incidental thereto. d) 1, 2 and 3

BIS has been providing traceable and tangible Ans) b benefits to the national economy in a number of Exp) Last year, the Government launched a new ways – providing safe reliable quality goods; unified ‘Jal Shakti’ ministry that is aimed at minimizing health hazards to consumers; providing clean drinking water as well as fighting India's water woes. promoting exports and import substitutes; control over proliferation of varieties etc. through

standardization, certification and testing. The new ministry was formed by merging the Ministry of Water Resources, River The activities of BIS can be broadly grouped Development and Ganga Rejuvenation and the Ministry of Drinking Water and under the following heads: 1. Standards formulation Sanitation. (Option b is correct.) 2. International activities

Source) 3. Product Certification 4. Hallmarking https://indianexpress.com/article/cities/delhi/bis- 5. Laboratory services prepares-draft-standard-for-water-supply-in- 6. Training services - National Institute of states-6564852/ & Training for Standardisation https://www.downtoearth.org.in/news/water/gov

7. Consumer Affairs and Publicity tracking purposes, sometimes replacing barcodes. NFC is still an emerging technology; RFID, The BIS sets standards for drinking water, toys – however, is currently in widespread use all over manufactured in India as well as imported. The the world. (Statement 1 is correct.) hallmarking of gold and silver jewellery is also done by the BIS. Hence, option d is correct. RFID tags contain an antenna and a memory chip that stores data. To see that data, you need Source) an RFID reader. These tags and readers are used https://indianexpress.com/article/cities/delhi/bis- in a mind-blowing array of applications. prepares-draft-standard-for-water-supply-in- states-6564852/ & https://bis.gov.in/ & The tags are embedded into retail products to help https://consumeraffairs.nic.in/organisation-and- stores keep tabs on inventory. Indeed, inventory units/division/bureau-indian-standards & and package tracking are two of the most https://indianexpress.com/article/india/from- common uses of RFID. But these tags can do september-1-imported-toys-to-be-let-in-only- much more. They're stuck under the dog's skin so after-quality-testing-paswan-6564851/ that the dog catcher can identify if the dog gets lost. The RFID highway toll tag in your car Q.8) With reference to the ‘Radio-Frequency automatically identifies you to the toll reader, Identification (RFID)’ and ‘Near Field even at top speed, which bills you later. Some Communication (NFC)’ technologies, which of airlines use RFID tags to efficiently track and the following statements is/are correct? control large loads of baggage. And RFID 1. Both employ radio signals for all sorts of appears in so-called smart passports and credit tagging and tracking purposes. cards, as well as identification badges that let 2. The working range of NFC technology is employees access secure areas. longer than that of the RFID technology. 3. The NFC technology allows one-way and RFID often works well at distances of many two-way communication whereas the RFID feet; otherwise, you'd have to veer your car technology allows only one-way dangerously close to a toll gate in order to make communication. sure the reader accepted your payment. And Select the correct answer using the code given RFID is a one-way communication system, in below. which data flows from tags to the reading a) 1 and 2 only equipment. b) 3 only c) 1 and 3 only NFC technology is a newer, more finely honed d) 1, 2 and 3 version of RFID. It operates at a maximum range of about 4 inches (10 centimetres) and Ans) c can be set up for one- or two-way Exp) Radio-Frequency Identification (RFID)’ communications. (Statement 2 is incorrect but and ‘Near Field Communication (NFC) statement 3 is correct.) NFC stands for near field communication, while RFID means radio frequency identification. Both Source) employ radio signals for all sorts of tagging and https://indianexpress.com/article/india/bihar-

elections-2020-guidelines-election-commission- activated until they find their specific antigen. In 6564805/ & general, there are three types of T cells: cytotoxic https://electronics.howstuffworks.com/differenc (Killer T-cells), helper, and regulatory. All of e-between-rfid-and-nfc.htm these must react to foreign antigens strongly to be effective for immunity. (Statement 2 is incorrect.) Q.9) Consider the following statements about T- cells, often seen in the news: Killer T-cells find and destroy infected cells that 1. They are a type of white blood cell. have been turned into virus-making factories. To 2. They originate in the thymus which is a do this they need to tell the difference between specialized primary lymphoid organ of the the infected cells and healthy cells with the help immune system. of special molecules called antigens. 3. They can cause inflammatory diseases. Source) Which of the statements given above is/are https://indianexpress.com/article/explained/new- correct? research-t-cell-response-in-covid-19-cases-even- a) 1 and 2 only where-no-antibodies-found/ & b) 1 and 3 only https://www.news-medical.net/health/What-are- c) 2 and 3 only T-Cells.aspx & https://askabiologist.asu.edu/t- d) 1, 2 and 3 cell

Ans) b Q.10) Consider the following statements: Exp) T-cells are a type of white blood cell that 1. The Election Commission of India imposes work with macrophages. Unlike macrophages limits on campaign expenditure incurred by a that can attack any invading cell or virus, each T- candidate, but not by a political party. cell can fight only one type of virus (pathogen). 2. The maximum limit of campaign expenditure You might think this means macrophages are in a Parliamentary constituency varies from State stronger than T-cells, but they aren’t. Instead, T- to State. cells are like a special forces unit that fights only Which of the statements given above is/are one kind of virus that might be attacking your correct? body. (Statement 1 is correct.) a) 1 only b) 2 only The most common context of T cells is within c) Both 1 and 2 infectious diseases, but they are used for other d) Neither 1 nor 2 aspects of adaptive immunity too. This includes responses to allergens and tumors. They maintain Ans) c immune homeostasis in humans over decades, Exp) Both statements 1 and 2 are correct. but can also be responsible for inflammatory The Election Commission (EC) imposes limits or autoimmune diseases. (Statement 3 is on campaign expenditure incurred by a correct.) candidate, not political parties. Expenditure by a Lok Sabha candidate is capped between Rs 50 T cells originate in the bone marrow but are lakh and Rs 70 lakh, depending on the state she matured in the thymus. However, they are not

is fighting from. In Assembly elections, the Narayan Bose was Aurobindo’s maternal ceiling is between Rs 20 lakh and Rs 28 lakh. This grandfather. includes money spent by a political party or a supporter towards the candidate’s campaign. At age four, he was sent to the Loreto Convent However, expenses incurred either by a party School in Darjeeling. Later, his father enrolled or the leader of a party for propagating the him at a public school in England. After school, party’s programme are not covered. he won a scholarship to King’s College, Cambridge, where he studied European classics. Candidates must mandatorily file a true account On his father’s insistence, he took the Indian Civil of election expenses with the EC. An incorrect Service Examination and passed in 1890 but account, or expenditure beyond the ceiling can failed in the horsemanship test due to which he attract disqualification for up to three years under could not enter the service. He returned to India Section 10A of The Representation of the People in 1893 and worked as the vice-principal of a Act, 1951. college in Baroda.

Source) After the declaration of the Partition of Bengal in https://indianexpress.com/article/opinion/column 1905, he shifted to the then Calcutta. There, he s/election-commission--elections-2020- joined active politics through non-cooperation coronavirus-pandemic-s-y-quraishi-6563324/ & movements and passive resistance. He formed http://employmentnews.gov.in/celling-of-poll- many secret revolutionary groups in Calcutta. He expenditure.pdf & inspired radical politicians such as Surendranath https://indianexpress.com/article/explained/electi Tagore, Jatindra Nath Banerjee and Jatindranath on-funding-poll-funding-election-commission- Mukherjee (Bagha Jatin) to join his group of bjp-on-election-expenditure-congress-on- rebels. He formed Anushilan Samiti, one of the election-funding-election-expenditure-election- first youth clubs in India, to protest against campaign-election-campaign-expenditure-bjp- imperial atrocities. election-exp/ The Bengal National College started Q.11) He was the first principal of the Bengal functioning from August 15, 1906 with Sri National College; was tried in the Alipore Bomb Aurobindo as its first Principal. During the Case; and wrote books Savitri and The Life 'Alipore Bomb Case' he resigned finally [1908] at Divine. He was the request of the College authorities. a) Bipin Chandra Pal b) Aurobindo Ghosh Ghosh was arrested in May 1908 on charge of c) Rabindranath Tagore being a part of Alipore Bomb Case, a bombing d) Satyendra Nath Bose incident in protest of the Bengal partition, which changed the course of Indian freedom movement. Ans) b His trial went on for a year, along with Exp) Aurobindo Ghosh: Born to surgeon Rashbihari Bose and other freedom fighters. Krishna Dhan Ghose and Swarnalata Devi in Chittaranjan Das stepped up as Ghose's defence Calcutta on August 15, 1871. Eminent writer Raj counsel in the trial.

He decided to give up active politics after his Select the correct answer using the code given stint in jail and devoted himself to below. spiritualism. He reached Pondicherry in 1910 a) 1 only and soon started getting followers and devotees b) 2 and 3 only who settled down at the place where Aurobindo c) 1 and 3 only lived and eventually a spiritual retreat (ashram) d) 1, 2 and 3 came up. Ans) b Years later, Ghose re-assembled his publications Exp) Election Commission - The Election and compiled them in a series of books. Some of Commission is a permanent and an independent these books were The Life Divine, Savitri, The body established by the Constitution of India Synthesis of Yoga, Essays on The Gita, The directly to ensure free and fair elections in the Secret of The Veda, Hymns to the Mystic Fire, country. Article 324 of the Constitution provides The Upanishads, The Renaissance in India, War that the power of superintendence, direction and and Self-determination, The Human Cycle, The control of elections to Parliament, State Ideal of Human Unity and The Future Poetry. Legislatures (Legislative Assembly and Legislative Council), the office of President of Hence, option b is correct. India and the office of Vice-President of India shall be vested in the Election Commission. Source) Thus, the Election Commission is an all-India https://indianexpress.com/article/lifestyle/life- body in the sense that it is common to both the style/sri-aurobindo-150-birth-anniversary- Central government and the state governments. webinar-celebrations-safic-6565704/ & Hence, option 1 is incorrect. https://www.hindustantimes.com/inspiring- lives/aurobindo-ghose-revolutionary-to- State Election Commission - The spiritualist/story- superintendence, direction and control of the fmEc8hAIsHBH3NQr1AkbBM.html & preparation of electoral rolls and the conduct of https://www.indiatoday.in/education-today/gk- all elections to the Panchayats and current-affairs/story/sri-aurobindo-facts-355680- Municipalities shall be vested in the State 2016-12-05 & Election Commission. It consists of a State http://www.sriaurobindoinstitute.org/saioc/Sri_ Election Commissioner to be appointed by the Aurobindo/bengal_national_college Governor. His conditions of service and tenure of office shall also be determined by the Governor. He shall not be removed from the office except in Q.12) With reference to a State with bicameral the manner and on the grounds prescribed for the legislature, which of the following elections removal of a judge of the state High Court. His is/are conducted by the State’s Election conditions of service shall not be varied to his Commission? disadvantage after his appointment. The state 1. Legislative Council legislature may make provision with respect to all 2. Panchayats matters relating to elections to the panchayats. 3. Municipalities Hence, options 2 and 3 are correct.

Source) (43 of 1951) and has secured at least one per https://indianexpress.com/article/india/rajiv- cent of the votes polled in the most recent Lok kumar-election-commissioner-ashok-lavasa- Sabha or State election will be allotted a verified 6564700/ & Indian Polity – M Laxmikanth, account by the Election Commission of India. Chapter – 42 ‘Election Commission’. Electoral bond transactions can be made only via this account. Hence, option b is correct. Q.13) Which one of the following statements is not correct regarding electoral bonds? The electoral bonds will not bear the name of the a) Only Reserve Bank of India and State Bank donor. In essence, the donor and the party details of India are allowed to sell these bonds for a will be available with the bank, but the political limited period of time per year. party might not be aware of who the donor is. The b) Only those registered political parties which intention is to ensure that all the donations made have secured at least one per cent of the votes to a party will be accounted for in the balance polled in the most recent Lok Sabha or State sheets without exposing the donor details to the election are eligible for funding through these public. bonds. c) It can be purchased by any citizen of India or Source) a body incorporated in India. https://indianexpress.com/article/india/govt- d) The bonds are similar to bank notes that are likely-to-allow-sale-of-electoral-bonds-in- payable to the bearer on demand and are free october-6564799/ & https://www.business- of interest. standard.com/about/what-is-electoral-bond & https://www.thehindu.com/news/national/the- hindu-explains-what-is-an-electoral-bond-and- Ans) a how-do-we-get-one/article22367124.ece Exp) Electoral Bonds - An electoral bond is designed to be a bearer instrument like a Q.14) The Reserve Bank of India (RBI) has Promissory Note — in effect, it will be similar to recently set up an expert committee under veteran a bank note that is payable to the bearer on banker K V Kamath (Kamath Committee) to demand and free of interest. It can be purchased a) review the Economic Capital Framework to by any citizen of India or a body incorporated determine the appropriate levels of reserve in India. Hence, options c and d are correct. the central bank should hold. b) suggest financial parameters for one-time The bonds will be issued in multiples of ₹1,000, restructuring of business loans. ₹10,000, ₹1 lakh, ₹10 lakh and ₹1 crore and will c) review the ATM Interchange Fee Structure be available at specified branches of State Bank with a view to give a fillip to the ATM of India. They can be bought by the donor with a deployment in the unbanked areas. KYC-compliant account. Donors can donate the d) examine the issues relating to the offshore bonds to their party of choice which can then be Rupee markets. cashed in via the party's verified account within 15 days. Hence, option a is incorrect. Ans) b Exp) The Reserve Bank of India (RBI) has Every party that is registered under section 29A recently set up an expert committee under veteran of the Representation of the Peoples Act, 1951 banker K V Kamath to suggest financial

parameters for resolution of coronavirus-related The Task Force was formed to examine the issues stressed assets. relating to the offshore Rupee markets in depth and recommend appropriate policy measures that In a major relief to corporate and retail also factored in the requirement of ensuring the borrowers, the RBI has permitted banks to go stability of the external value of the Rupee. It was for one-time restructuring of loans that are chaired by Smt. Usha Thorat. facing stress due to the COVID-19 crisis with a view to mitigating risks to financial stability. Committee to Review the ATM Interchange Fee Structure The restructuring will be allowed as per the Reserve Bank of India constituted a Committee prudential framework issued on June 7, 2019. to Review the ATM Interchange Fee Structure And as a special window under the Prudential with a view to give a fillip to the ATM Framework on Resolution of Stressed Assets deployment in the unbanked areas. It was headed issued on June 7, 2019, the RBI announced a by Shri V. G. Kannan. ‘Resolution Framework for COVID-19-related Stress’. Source) https://indianexpress.com/article/business/banki The resolution framework envisages constitution ng-and-finance/rbi-sets-up-panel-under-k-v- of an expert committee by the RBI to make kamath-to-suggest-steps-to-deal-with- recommendations on the required financial coronavirus-related-stressed-assets-6544280/ & parameters to be factored in into the https://www.business- resolution standard.com/article/current-affairs/kamath- plans, with sector-specific benchmark ranges for committee-recommendations-will-be-out-by- such parameters. september-6-rbi-governor- 120082100637_1.html & The expert committee will also undertake the https://www.livemint.com/industry/banking/debt process validation for the resolution plans to be -recast-rbi-announces-names-of-members-of-kv- implemented under this framework, without kamath-committee-11596788101877.html going into the commercial aspects, in respect of all accounts with an aggregate exposure of Rs 1,500 crore and above at the time of Q.15) With reference to the Active invocation. Hence, option b is correct. Pharmaceutical Ingredients (APIs), consider the following statements: Economic Capital Framework (ECF) 1. Chinese manufacturers make around 40% of Committee all APIs used worldwide. The Reserve Bank of India constituted the expert 2. Indian drug-makers import around 70 % of committee to review the Economic Capital their total API requirements from China. Framework (ECF) to determine the appropriate levels of reserve the central bank should hold. It Which of the statements given above is/are was chaired by Dr. Bimal Jalan. correct? a) 1 only Task Force on Offshore Rupee Markets b) 2 only c) Both 1 and 2

d) Neither 1 nor 2 all APIs used worldwide. (Statement 1 is correct.) Ans) c Exp) Active pharmaceutical ingredient (API) - Despite being a leading supplier of high-quality Active pharmaceutical ingredient (API), is the medicines to several countries, Indian term used to refer to the biologically active pharmaceutical industry is highly dependent on component of a drug product (e.g. tablet, China for raw materials even to produce capsule). Drug products are usually composed of medicines as basic as Crocin. Indian drug-makers several components. The aforementioned API is import around 70 per cent of their total bulk the primary ingredient. Other ingredients are drug requirements from China. (Statement 2 is commonly known as "excipients" and these correct.) substances are always required to be biologically safe, often making up a variable fraction of the Source) drug product. The procedure for optimizing and https://indianexpress.com/article/business/bulk- compositing this mixture of components used in drugs-medical-devices-production-scheme- the drug is known as "formulation." These raw norms-for-companies-announced-6526443/ & materials are also known as bulk drugs. https://www.cphi.com/content/dam/Informa/cphi /europe/en/2020/pdf-files/API- For example, if the API is a solid and the drug is CPhI_Zone_Article_0.pdf & required to have a liquid dosage form, such as a https://theprint.in/theprint-essential/what-are- cough syrup, then the excipients would be the apis-and-how-they-threaten--status-of-a- liquids that are used to formulate the syrup. The pharmacy-to-the-world/370941/ & design criteria for any small molecule API is https://www.sciencedaily.com/releases/2014/06/ usually a combination of several factors that goes 140605093305.htm#:~:text=Active%20pharmac beyond the intended therapeutic effect, and eutical%20ingredient%20(API)%2C,(e.g.%20ta usually heavily encompasses both blet%2C%20capsule).&text=Other%20ingredie pharmacokinetic and pharmacodynamic nts%20are%20commonly%20known,fraction%2 considerations, so for this reason, API molecules 0of%20the%20drug%20product. have many chemical functional groups. Q.16) With reference to the Jal Jeevan Mission The current global API supplier base consists of (JJM), consider the following statements: hubs in which manufacturers specialize in 1. It aims to provide all households in India with producing different types of ingredients for safe and adequate water through household separate sections of the pharmaceutical market. tap connections by 2024. China, for example, has a reputation for the 2. It is a central sector scheme funded production of low cost, high volume ingredients. completely from central funds. And it is a major global source for the drug industry. Data compiled by UK regulator, the Which of the statements given above is/are Medicines and Healthcare Products correct? Regulatory Agency (MHRA), suggests a) 1 only Chinese manufacturers make around 40% of b) 2 only c) Both 1 and 2 d) Neither 1 nor 2

Ans) d Q.17) Which of the following rights have been Exp) Jal Jeevan Mission - Jal Jeevan Mission, is derived from the Right to Freedom of Speech and envisioned to provide safe and adequate Expression under Article 19 of the Constitution? drinking water through individual household 1. Right to propagate one’s views as well as tap connections by 2024 to all households in views of others rural India. The programme will also implement 2. Freedom of commercial advertisements source sustainability measures as mandatory 3. Right against tapping of telephonic elements, such as recharge and reuse through conversation grey water management, water conservation, rain 4. Right to strike water harvesting. The Jal Jeevan Mission will be Select the correct answer using the code given based on a community approach to water and will below. include extensive Information, Education and a) 1 and 2 only communication as a key component of the b) 2 and 4 only mission. The Mission was launched on August c) 1, 2 and 3 only 15, 2019. It provides functional household tap d) 1, 2, 3 and 4 connections to all rural households not just BPL households. Hence, statement 1 is Ans) c incorrect. Exp) Freedom of Speech and Expression It implies that every citizen has the right to The goal of JJM is to provide functional express his views, opinions, belief and household tap connection to every household convictions freely by word of mouth, writing, with service level at the rate of 55 litres per printing, picturing or in any other manner. The capita per day (lpcd). Supreme Court held that the freedom of speech and expression includes the following: Financial Implication and Funding Pattern a) Right to propagate one’s views as well as under JJM: Total estimated cost of JJM is Rs. views of others. 3.60 Lakh Crore. The fund sharing pattern b) Freedom of the press. between Centre and State is 90:10 for Himalayan c) Freedom of commercial advertisements. (Uttarakhand, Himachal Pradesh) and North- d) Right against tapping of telephonic Eastern States, 100:0 for UTs and 50:50 for rest conversation. of the States. Hence, statement 2 is incorrect. e) Right to telecast, that is, the government has no monopoly on electronic media. Source) f) Right against bandh called by a political party https://indianexpress.com/article/explained/expla or organisation. ined-understanding-bis-daft-standard-for- g) Right to know about government activities. drinking-water-supply-6565866/ & h) Freedom of silence. https://jaljeevanmission.gov.in/content/about- i) Right against imposition of pre-censorship on jjm & https://vikaspedia.in/health/sanitation-and- a newspaper. hygiene/jal-jeevan-mission & j) Right to demonstration or picketing but not https://www.business-standard.com/about/what- right to strike. is-jal-jeevan-mission & https://jalshakti- ddws.gov.in/sites/default/files/JJM_note.pdf Hence, option c is correct.

is deemed to arise in India. Income earned as The State can impose reasonable restrictions on interest, royalties, dividends, technical the exercise of the freedom of speech and services fees, or gains through the sale of expression on the grounds of sovereignty and assets based in India is taxable. Corporate tax integrity of India, security of the state, friendly also includes the following: relations with foreign states, public order, • Minimum Alternate Tax (MAT) - Levied decency or morality, contempt of court, on zero tax companies whose accounts are defamation, and incitement to an offence. prepared as per the guidelines of the Companies Act. (Option 1 is correct.) Source) • Fringe Benefits Tax - Such direct tax is paid https://indianexpress.com/article/india/10-free- by companies on fringe benefits (drivers, speech-cases-this-year-in-supreme-court- maids, etc.) provided to employees. 6564796/ & Indian Polity – M Laxmikanth, • Dividend Distribution Tax (DDT) - This Chapter – 7 ‘Fundamental Rights’. tax is levied on any amounts that are declared, distributed, or paid by domestic Q.18) Consider the following: entities as dividends to the shareholders; 1. Minimum Alternate Tax foreign companies are exempt from DDT. 2. Dividend Distribution Tax (Option 2 is correct.) 3. Customs Duty • Securities Transaction Tax (STT) - This 4. Capital Gains Tax liability arises from income earned through Which of the taxes given above fall under the taxable securities transactions. category of direct taxes? a) 1, 2 and 4 only b) 2, 3 and 4 only 2. Wealth tax - This liability arises from the c) 1 and 3 only ownership of properties and is paid every d) 1, 2, 3 and 4 year based on the market value of the property. Property owners must pay this tax Ans) a irrespective of whether the property earns Exp) The Government of India levies two types them any income or not. of taxes on the citizens of India – Direct Tax and Indirect Tax. 3. Capital gains tax - This type of direct tax in India is payable on income earned from the Direct Tax - As the name suggests these taxes are sale of investments or assets. Capital assets directly paid by the assesse to the government. include investments in homes, art, These are not paid on behalf of the taxpayers but businesses, shares, bonds, and farms. Capital are imposed directly by the regulator. gains are calculated as the difference between Furthermore, this liability is non-transferable to the sale value and the purchase value of these another taxpayer. assets. This tax is classified as either short- 1. Corporate tax - Such tax is levied on term or long-term based on its holding domestic companies that are different from period. All assets (except securities) that are the shareholders. This tax is also payable by sold within 36 months of acquisition are foreign corporations whose income arises or liable to short-term gains. Gains made through the sale of assets that are held for

more than 36 months are long-term assets. 2. Value-added tax - It is a tax on the (Option 4 is correct.) consumption of a product, and it is levied on its value when additional or margin value is Indirect Tax - Indirect taxes are not levied added to the product during the subsequent directly on the income of an individual. Rather, stages of production, starting from the point they are levied through a channel of of production to the point of sale. The tax consumption. These taxes are usually levied on goes on adding at every stage of production sellers of goods and services who pass on the tax as per the value addition. burden to the buyers. The government collects the tax from the seller(s). Since the end consumer 3. Service tax - Service tax is a tax levied on the directly does not pay the tax to the government, service providers. It is levied on services like hence, it is an indirect tax. air-conditioned restaurants, accommodation provided by the hotel, inns, services provided Recent developments by travel agents, cab services, cable services, In India, a consumer earlier paid several indirect etc. The idea behind service tax is similar to taxes including sales tax, services tax, central that of excise duty. The incidence of this tax excise duty, additional customs duty, state-level applies to the disbursement of services. value added tax and octroi tax, among others. With the implementation of the Goods and Customs Duty - It is an Import duty levied on Services Tax (GST) from 1st July 2017, almost goods coming from outside the country, all the indirect taxes have been subsumed under ultimately paid by consumers and retailers in GST. Now, all these taxes are paid as one tax India. (Option 3 is incorrect.) called the GST. Source) However, there are some indirect taxes which https://indianexpress.com/article/business/econo have not been brought under the radar of GST. my/direct-tax-refunds-worth-rs-88652-cr-issued- Customs duty, security transaction tax, building to-over-24-lakh-taxpayers-i-t-dept-6564815/ & and welfare cess and electricity duty levied by https://cleartax.in/s/direct-indirect-taxation- local authorities and state governments are some india-explained of them. & https://www.hdfclife.com/insurance- knowledge-centre/tax-saving-insurance/direct- Below is a list of indirect taxes that are levied on tax#:~:text=Direct%20taxes%20are%20levied% the supply of good and services in India, which 20on,majority%20of%20the%20Indian%20popu now is taxed under the common name of GST. lation. & 1. Excise duty - Excise duty is levied on goods https://www.adityabirlacapital.com/abc-of- and services that are produced in India. This money/what-are-the-kinds-of-indirect-taxes-in- is a tax on manufacturing which is paid by the india#:~:text=There%20are%20two%20types% manufacturer to the government. The 20of,through%20a%20channel%20of%20consu manufacturer distributes and passes this tax mption. on the end consumers.

Q.19) In recent times, the bond yields in India Corporations also issue bonds to fund expansions have increased. It indicates that and projects, or tide over budgetary deficits. 1. the market price of the bonds have increased. Bonds issued by corporations are generally safer 2. the company or government that issued it is than equities, but some bonds offer an option to financially stressed and may not be able to convert the instrument into stocks. Non- repay the capital. convertible corporate bonds offer coupons and Which of the statements given above is/are function like any other bond. correct? a) 1 only How do bonds work? b) 2 only Consider a bond that is issued by a company or c) Both 1 and 2 government for Rs.100 with a coupon of 7% for d) Neither 1 nor 2 a period of five years. This implies that every year, the issuer will pay interest of 7%, while Ans) b the principal will be refunded after the bond Exp) What are bonds? matures. What drives the bond market is the fact A bond is a debt instrument which acts as an that bondholders are free to sell their bonds IOU(an informal document acknowledging debt). before maturity. It can be traded in financial markets like equities and other commodities. It is commonly The face value of a bond is what it sold for used by governments to raise capital in order to initially. Since they are transferable through sale fund domestic growth and development projects. in the bond market, their value fluctuates. The Investors take on government debt, and in return, returns accrued by holders is measured by the are assured a stream of revenue for the duration yield, which is the rate of interest paid as of the time it takes the bond to mature. Bonds coupons. issue coupons, which are interest payments made in part to the repayment of the capital that was Yield is calculated as the coupon divided by the borrowed. The final payment is made when the value of the bond. It is multiplied by 100 to be bond attains maturity. However, governments are expressed in percentage. In effect, yield = not the only entities issuing bonds. (coupon/value)*100)

Who buys bonds and why? In the example considered above, if the face Unlike equities, which are vulnerable to the value of the bond fell to Rs.80, the yield would vagaries of the stock market, investing in bonds rise to 8.75%, that is (7/80)*100). is a relatively safer proposition since the capital Correspondingly, if the value rose to Rs.120, the is returned on maturity. The downside is that the yield would drop to 5.83%, that is (7/120)*100). returns on bonds may not be comparable to those of equities. Government bonds, also known as G- It should be noted that even if the value of a bond Sec, are issued by governments with maturity fluctuates, the amount that can be redeemed after terms ranging from medium to long term. the term of maturity is reached is the face value, which is Rs.100 in this case. Bond yield is Do only governments issue bonds? inversely proportional to its current value. The

greater the yield, the lower the current market Which of the statements given above is/are price of the bond. (Statement 1 is incorrect.) correct? a) 1 and 2 only Bond yields as an economic indicator b) 2 only Conventional metrics used by economists to c) 1 and 3 only diagnose the health of a country's economy d) 3 only include inflation, lending rate of the central bank, growth rate, and national income. However, bond Ans) b yields are also a very prescient means of gauging Exp) Indian National Congress the trajectory of an economy. As investors sell In the later 1870s and early 1880s, a solid ground government bonds, prices drop, and yields had been prepared for the establishment of an all- increase. A higher yield indicates greater risk. India organisation. The final shape to this idea If the yield offered by a bond is much higher than was given by a retired English civil servant, A.O. what it was when issued, there is a chance that the Hume, who mobilised leading intellectuals of the company or government that issued it is time and, with their cooperation, organised the financially stressed and may not be able to repay first session of the Indian National Congress at the capital. (Statement 2 is correct.) Gokuldas Tejpal Sanskrit College in Bombay in December 1885. As a prelude to this, two Enhanced risk with such bonds is offset by sessions of the Indian National Conference had greater returns, adding to their appeal. been held in 1883 and 1885, which had representatives drawn from all major towns of Source) India. Surendranath Banerjea and Ananda Mohan https://www.livemint.com/market/stock-market- Bose were the main architects of the Indian news/rbi-sends-signal-to-traders-that-bond- National Conference. The first session of INC yields-may-head-higher-11598236125528.html was attended by 72 delegates and presided & over by Womesh Chandra Bannerjee. https://www.thehindu.com/business/Economy/w (Statement 1 is incorrect.) hat-rising-bond-yields-tell-us-about-the- economy/article23426821.ece Moderate Approach The moderate political activity involved Q.20) With reference to the British colonial rule constitutional agitation within the confines of law in India, consider the following statements and showed a slow but orderly political progress. regarding the Indian National Congress (INC): The Moderates believed that the British 1. The first session of INC was organized in basically wanted to be just to the Indians but Calcutta under the Presidentship of Womesh were not aware of the real conditions. Chandra Bannerjee. Therefore, if public opinion could be created in 2. The Moderates believed that the British the country and public demands be presented to basically wanted to be just to Indians but the government through resolutions, petitions, were not aware of real conditions. meetings, etc., the authorities would concede 3. Sardar Vallabhbhai Patel never presided over these demands gradually. (Statement 2 is any session of the INC. correct.)

To achieve these ends, they worked on a two- d) Neither 1 nor 2 pronged methodology—one, create a strong public opinion to arouse consciousness and Ans) b national spirit and then educate and unite people Exp) NITI Aayog recently in the partnership on common political questions; and two, with the Institute for Competitiveness released persuade the British Government and British the first Export Preparedness Index 2020. public opinion to introduce reforms in India on (Statement 2 is incorrect.) the lines laid out by the nationalists. They used the method of ‘prayer and petition’ and if that Gujarat topped the ranking among the states for failed, they resorted to constitutional agitation. their potential for exports and performance in the overall ranking. It indicates the presence of basic Sardar Vallabhbhai Patel was president of the facilities, a conducive environment, and the reach Karachi session of the INC in 1931. The of the exports’ footprint in the states. (Statement conference endorsed the Gandhi-Irwin Pact under 1 is correct.) his presidentship at this session. (Statement 3 is incorrect.)

Source) https://indianexpress.com/article/india/congress- working-committee-meeting-resolution-full- text-sonia-gandhi-6568139/ & A Brief History of Modern India – Rajiv Ahir,

Chapter – 11 ‘Indian National Congress: At present, 70 per cent of India’s export has been Foundation and the Moderate Phase’ & dominated by five states – Maharashtra, Gujarat, https://indianexpress.com/article/india/here-is-a- Karnataka, Tamil Nadu and Telangana, the list-of-past-presidents-of-indian-national- government’s report highlighted. congress-4967084/ Source) Q.21) With reference to the Export Preparedness https://indianexpress.com/article/business/states- Index (EPI) 2020, consider the following failed-to-address-three-main-issues-niti-report- statements: on-export-preparedness-6571360/ & 1. Gujarat was adjudged the best state in the https://niti.gov.in/sites/default/files/2020- overall ranking among the states for their 08/Digital_ExportPreparednessIndex2020_0.pdf potential for exports and performance. 2. It has been released by the Ministry of Q.22) Which one of the following is the best Commerce in partnership with the Institute description of ‘INS Viraat’, that was in the news for Competitiveness. recently? Which of the statements given above is/are not a) Amphibious warfare ship correct? b) Nuclear powered submarine a) 1 only c) Torpedo launch and recovery vessel b) 2 only d) Aircraft carrier c) Both 1 and 2

Ans) d Sin goods are goods which are considered Exp) INS Viraat was a Centaur-class aircraft harmful to society. Example of sin goods: carrier of the Indian Navy. INS Viraat was the Alcohol and Tobacco, Candies, Drugs, Soft flagship of the Indian Navy before INS drinks, Fast foods, Coffee, Sugar, Gambling and Vikramaditya was commissioned in 2013. Pornography.

Originally commissioned by the British Navy as Taxes levied by the government on sin goods HMS Hermes on November 18, 1959, the called Sin Tax. A sin tax is an excise (now GST) aircraft carrier had taken part in the Falkland tax specifically levied on certain goods deemed Islands war in 1982. India bought the British harmful to society, for example alcohol and carrier in 1986 and rechristened it as INS Viraat. tobacco, candies, drugs, soft drinks, fast foods, Under the Indian flag, the ship that could carry 24 coffee, sugar, gambling and pornography. Sin aircraft, including fighter jets and helicopters, taxes seek to deter people from engaging in clocked more than 22,622 flying hours and spent socially harmful activities and behaviours, but 2,252 days at sea sailing 5.88 lakh nautical miles, they also provide a source of revenue for which is almost equivalent to covering the globe governments. (Option a is correct.) 27 times. It was decommissioned in March 2017. Sin taxes are typically added to liquor, cigarettes, and goods that are considered morally hazardous. This is the second aircraft carrier to be broken in Because they generate enormous revenue, state India in the past six years. In 2014, INS Vikrant, governments favour sin taxes. Society accepts sin which played a role in the historic 1971 war with taxes because they affect only those who use sin Pakistan was broken down in Mumbai. taxed products or engage in sin taxed behaviors. When individual states run a deficit, a sin tax is Source) generally one of the first taxes recommended by https://indianexpress.com/article/india/ins- lawmakers to help fill the budget gap. viraat-to-be-broken-at-alang-and-sold-as-scrap- 6568260/ & A sin tax is a type of Pigovian tax, which is levied https://en.wikipedia.org/wiki/INS_Viraat on companies which create negative externalities with their business practices. Sin tax proponents Q.23) With reference to Indian economy, ‘sin maintain that the targeted behaviors and goods tax’ is produce negative externalities. In other words, a) a tax specifically levied on certain goods they foist an unfair burden on the rest of society. deemed harmful to society. The effects of alcohol and tobacco products b) a religious tax imposed by the State on increase health care costs driving up the cost of sinners as per their religious beliefs. insurance for everyone. Also, compulsive c) a tax specifically levied on certain people gambling compromises the security and well- convicted of heinous offences. being of stable home life, children, and families d) a tax imposed by religious leaders on their of the gambler. followers if they break any religious code. One purpose of a Pigovian tax is to create an Ans) a incentive to reduce negative externalities. The sin Exp) Sin Tax

tax seeks to reduce or eliminate consumption of Warli Folk Paintings - Maharashtra is known harmful products by making them more for its Warli folk paintings. Warli is the name of expensive to obtain. the largest tribe found on the northern outskirts of Mumbai, in Western India. Despite being in Source) such close proximity to the largest metropolis in https://www.thehindu.com/business/two- India, Warli tribesmen shun all influences of wheelers-neither-luxury-nor-sin-goods- modern urbanization. Warli Art was first fm/article32440803.ece & discovered in the early seventies. While there are https://www.investopedia.com/terms/s/sin_tax.a no records of the exact origins of this art, its roots sp & https://howtoexportimport.com/Meaning- may be traced to as early as the 10th century A.D. of-Sin-Goods-Sin-Tax-9384.aspx Warli is the vivid expression of daily and social events of the Warli tribe of Maharashtra, used by Q.24) Consider the following pairs: them to embellish the walls of village houses. Paintings State This was the only means of transmitting folklore 1. Cheriyal Paintings : Telangana to a populace not acquainted with the written 2. Warli Paintings : Rajasthan word. This art form is simple in comparison to the 3. Saora Paintings : Odisha vibrant paintings of Madhubani. Which of the pairs given above is/are correctly matched? Women are mainly engaged in the creation of a) 1 and 2 only these paintings. These paintings do not depict b) 2 only mythological characters or images of deities, c) 1 and 3 only but depict social life. Images of human beings d) 3 only and animals, along with scenes from daily life are created in a loose rhythmic pattern. These tribal Ans) c paintings of Maharashtra are traditionally done in Exp) Cheriyal Scroll Paintings - Cheriyal Scroll the homes of the Warlis. Painted white on mud Painting is a popular and modified version of walls, they are pretty close to prehistoric cave Nakashi art, considered highly rich in the local paintings in execution and usually depict scenes motifs. This art form is unique to the state of of human figures engaged in activities like Telangana and made mostly in Hyderabad hunting, dancing, sowing and harvesting. (Pair 2 currently. is not correctly matched.)

These scrolls are painted in narrative format Saora Paintings - Saora paintings have their similar to a film roll or even comic strips, and origin in the mud walls of aboriginal Lanjia depict stories from the Indian mythology as well Saora tribal homes in Odisha. Also called the as the shorter stories related to the Puranas and hill Saoras, the community inhabits the remote Epics. Scroll paintings are known for their rich ranges flanking the great Bansadhara river in history and they also assume a significant role southern Odisha. among Asia’s artistic traditions. The Cheriyal paintings represent a distinct local invention, A Saora painting is called Idital and the person based mainly on local traditions. (Pair 1 is who creates it is known as the iditalmar. correctly matched.) Interesting anecdotes are associated with their art

practice. Iditals are sketched to appease Saora What does Nuakhai mean? ancestors and deities that may have caused Nuakhai is a combination of two words that diseases faced by the iditalmars or the villagers at signifies eating of new rice as 'nua' means new large. In Saora society, a shaman is believed to be and 'khai' means eat. an intermediary between the worlds of the living and the dead. The iditalmar draws instructions On this day, people worship food grain and from the shaman. (Pair 3 is correctly matched.) prepare special meals. Farmers offer the first produce from their lands to Goddess Source) Samaleswari, the famous 'Mother Goddess' of https://indianexpress.com/article/express- Sambalpur district of Odisha. sunday-eye/how-face-masks-are-redefining- wearable-art-6534452/ & Source) https://www.thehindu.com/entertainment/art/sao https://www.livemint.com/news/india/pm-modi- ra-paintings-travel-from-tribal-homes-to-living- extends-greeting-to-farmers-on-nuakhai-juhar- rooms-overseas/article23343798.ece & 11598157982874.html https://telanganatourism.gov.in/partials/about/art s-crafts/cheriyal-scroll-paintings.html & Q.26) Consider the following statements: https://asiainch.org/craft/saora-folk-painting-of- 1. Chit funds are savings financial instruments. odisha/ & https://knowindia.gov.in/culture-and- 2. Chit funds are credit financial instruments. heritage/folk-and-tribal-art/warli-folk- 3. Chit funds are regulated by the Reserve Bank painting.php of India. Which of the statements given above is/are Q.25) With reference to India’s culture and correct? tradition, what is ‘Nuakhai Juhar’? a) 1 and 3 only a) It is an ancient Bhakti cult of Shaivism still b) 2 and 3 only prevalent in some parts of South India. c) 3 only b) It is one of the most ancient agricultural d) 1 and 2 only festivals celebrated in Odisha, Chhattisgarh and areas of neighbouring states. Ans) d c) It is an ancient form of dance-drama and a Exp) What is a chit fund? living tradition in the northern part of A chit fund is both a savings and credit product. Malabar. It bears a predetermined value and is of a fixed d) It is an ancient martial art and a living duration, mostly two to three years. Each tradition in some parts of South India. scheme admits a specific number of members whose monthly contributions add up to the total Ans) b value of the chit fund at the end of the term. Exp) What is Nuakhai Juhar? Nuakhai Juhar, one of the most ancient festivals, Each subscriber is entitled to a prize amount is the agricultural festival celebrated in determined by lot, auction or tender depending on Odisha, Chhattisgarh and areas of neighbouring the nature of the chit fund. Typically the prize states to welcome the new crop of the season. It amount is the entire pool of contribution minus a is also called Nuakhai Parab or Nuakahi discount which is redistributed to subscribers as a Bhetghat. (Option b is correct.)

dividend. For example, consider an auction-type • The capital requirement for establishing chit chit fund with 50 subscribers contributing Rs 100 funds is Rs 1 lakh and at least 10% of profits every month. The monthly pool is Rs 5,000 and should be transferred to a reserve fund. this is auctioned out every month. The winning • The amount of discount (i.e. the bid) is bid, say Rs 1000, would be the discount and be capped at 40% of the total chit fund value. distributed among the subscribers. The winning • States may appoint a Registrar who would be bidder would then receive Rs 4,000 (Rs 5,000 – responsible for regulation, inspection and 1,000) while the rest of subscribers would receive dispute settlement in the sector. Rs 20 (1000/50). Winners cannot enter the • Any grievances over decisions made by the auction again and will be liable for the monthly Registrar can be subject to appeals directed to subscription as the process is repeated for the the state government. duration of the scheme. The company managing • Chit fund managers are required to deposit the chit fund (foreman) would retain a the entire value of the chit fund (can be done commission from the prize amount every month. in 50% cash and 50% bank guarantee) with Collectively, the subscribers to a chit fund are the Registrar for the duration of the chit referred to as a chit group and a chit fund cycle. company may run many such groups. • Prize Chits and Money Circulation Schemes (Banning) Act, 1978 The Prize Chits and What are the laws governing chit funds? Money Circulation Schemes (Banning) Act, Classifying them as contracts, the Supreme 1978 defines and prohibits any illegal chit Court has read chit funds as being part of the fund schemes (e.g. schemes where auction Concurrent List of the Indian Constitution; winners are not liable to future payments). hence both the centre and state can frame • Again, the responsibility for enforcing the legislation regarding chit funds. States like Tamil provisions of this Act lies with the state Nadu, Andhra Pradesh and Kerala had enacted government. legislation (e.g The Kerala Chitties Act, 1975 and The Tamil Nadu Chit Funds Act, 1961) for What is the role of RBI and SEBI? regulating chit funds. The Reserve Bank of India (RBI) is the regulator for banks and other non-banking Chit Funds Act, 1982 - In 1982, the Ministry of financial companies (NBFCs) but does not Finance enacted the Chit Funds Act to regulate regulate the chit fund business. While chit the sector. Under the Act, the central government funds accept deposits, the term ‘deposit’ as can choose to notify the Act in different states on defined under the Reserve Bank of India Act, different dates; if the Act is notified in a state, 1934 does not include subscriptions to chits. then the state act would be repealed. States are However the RBI can provide guidance to state responsible for notifying rules and have the governments on regulatory aspects like creating power to exempt certain chit funds from the rules or exempting certain chit funds. provisions of the Act. The central government notified the Act in Arunachal Pradesh, Gujarat, As the regulator of the securities market, SEBI Haryana, Kerala and Nagaland. Under the Act, all regulates collective investment schemes. But the chit funds require previous sanction from the SEBI Act, 1992 specifically excludes chit funds state government. from their definition of collective investment

schemes. In the recent case with Saradha Group, The Agricultural and Processed Food Products the SEBI investigation discovered that Sarada Export Development Authority (APEDA) was were, in effect, operating a collective investment established by the Government of India under scheme without SEBI’s approval. the Agricultural and Processed Food Products Export Development Authority Act passed by Source) the Parliament in December, 1985. The https://indianexpress.com/article/cities/delhi/fam Authority replaced the Processed Food Export ily-running-chit-fund-dupes-dozens-savings- Promotion Council (PFEPC). 6566019/ & https://economictimes.indiatimes.com/wealth/in Functions vest/chit-funds-how-it-works-red-flags-and- In accordance with the Agricultural and your-rights-as-an- Processed Food Products Export Development investor/articleshow/73805938.cms & Authority Act, 1985, (2 of 1986) the following https://www.bankbazaar.com/recurring- functions have been assigned to the Authority. deposit/recurring-deposit-vs-chit-fund.html & • Development of industries relating to the https://www.prsindia.org/theprsblog/chit-funds- scheduled products for export by way of q providing financial assistance or otherwise for undertaking surveys and feasibility Q.27) Which of the following is/are the functions studies, participation in enquiry capital of the Agricultural and Processed Food Products through joint ventures and other reliefs and Export Development Authority (APEDA)? subsidy schemes; 1. Registration of persons as exporters of the • Registration of persons as exporters of the scheduled products. scheduled products on payment of such 2. Fixing of standards and specifications for the fees as may be prescribed; (Statement 1 is scheduled products for the purpose of correct.) exports. • Fixing of standards and specifications for 3. Improving packaging and marketing of the the scheduled products for the purpose of Scheduled products. exports; (Statement 2 is correct.) 4. Carrying out an inspection of products for the • Carrying out inspection of meat and meat purpose of ensuring the quality of such products in slaughterhouses, processing products. plants, storage premises, conveyances or Select the correct answer using the code given other places where such products are kept or below. handled for the purpose of ensuring the a) 1 and 3 only quality of such products; (Statement 4 is b) 1, 3 and 4 only correct.) c) 2 and 4 only • Improving of packaging of the Scheduled d) 1, 2, 3 and 4 products; (Statement 3 is correct.) • Improving of marketing of the Scheduled Ans) d products outside India; Exp) Agricultural and Processed Food • Promotion of export oriented production and Products Export Development Authority development of the Scheduled products; (APEDA)

• Collection of statistics from the owners of the Shompen, and the Nicobarese, the factories or establishments engaged in the Sentinelese. production, processing, packaging, marketing or export of the scheduled The Andamans is home to five PVTGs, the products or from such other persons as may Sentinelese, Jarawa, Great Andamanese, be prescribed on any matter relating to the Onge and Shompen. This is besides the scheduled products and publication of the Nicobarese, who are Scheduled Tribes. statistics so collected or of any portions (Statement 1 is incorrect.) thereof or extracts therefrom; • Training in various aspects of the industries Amit Kumar Ghosh, the Superintending connected with the scheduled products; Anthropologist at the Anthropological Survey of • Such other matters as may be prescribed. India, pointed out that in the 1850s, the Great Andamanese numbered between 5,000 and 8,000. “Then a penal colony was set up and diseases like Source) syphilis, gonorrhoea, flu and others spread. By https://www.newindianexpress.com/business/20 1901, their population had dropped to 625, and by 20/aug/18/indias-agriculture-exports-up-2324- the 1931 Census, only 90 Great Andamanese per-cent-to-over-rs-25500-crore-in-march-june- were left. By the 1960s, they were down to a mere government-2185057.html & 19, and were settled on Strait Island,” he said, https://apeda.gov.in/apedawebsite/about_apeda/ highlighting the vulnerability of the PVTGs to About_apeda.htm & https://cleartax.in/s/apeda- illnesses. registration Ghosh added that the danger is even higher for Q.28) Consider the following statements: other tribes. “The Great Andamanese have 1. All the tribes of Andaman and Nicobar been in contact with outsiders for the last 50 Islands have been categorised as Particularly years. But a disease like this could wipe out the Vulnerable Tribal Groups. entire population of the Jarawas and Sentinelese.” 2. The Great Andamanese is the only tribe of the (Statement 2 is incorrect.) Andaman and Nicobar Islands which still does not have any contact with the outside The Sentinelese are the most recluse of them, world. resisting outside contact. They don’t have any Which of the statements given above is/are contact with the outside world. correct? a) 1 only Source) b) 2 only https://indianexpress.com/article/india/four- c) Both 1 and 2 members-of-great-andamenese-tribe-test- d) Neither 1 nor 2 positive-for-covid-19-6572424/ & https://indianexpress.com/article/india/andaman- Ans) d tribe-coronavirus-covid-19-6572841/ & Exp) There are six native tribal groups in the https://northmiddle.andaman.nic.in/people- Andaman and Nicobar Islands. They are - the cultures/ Great Andamanese, the Jarawas, the Onge,

Q.29) With reference to the Article 15 of the There are three exceptions to this general rule Constitution, the State is empowered to reserve of non-discrimination: seats for any socially and educationally backward • The state is permitted to make any special classes of citizens in provision for women and children. For 1. Public Educational Institutions example, reservation of seats for women in 2. Private Educational Institutions aided by the local bodies or provision of free education for State children. 3. Private Educational Institutions unaided by • The state is permitted to make any special the State provision for the advancement of any socially 4. Minority Educational Institutional and educationally backward classes of citizens or for the scheduled castes and Which of the statements given above is/are scheduled tribes. For example, reservation of correct? seats or fee concessions in public a) 1 and 2 only educational institutions. (Statement 1 is b) 1 and 4 only correct.) c) 1, 2 and 3 only • The state is empowered to make any special d) 1, 2 and 4 only provision for the advancement of any socially

and educationally backward classes of Ans) c citizens or for the scheduled castes or the Exp) Article 15 provides that the State shall not scheduled tribes regarding their admission to discriminate against any citizen on grounds only educational institutions including private of religion, race, caste, sex or place of birth. The educational institutions, whether aided or two crucial words in this provision are unaided by the state, except the minority ‘discrimination’ and ‘only’. The word educational institutions. (Statements 2 and ‘discrimination’ means ‘to make an adverse 3 are correct but statement 4 is incorrect.) distinction with regard to’ or ‘to distinguish unfavourably from others’. The use of the word Source) ‘only’ connotes that discrimination on other https://indianexpress.com/article/india/sc-says- grounds is not prohibited. 2004-verdict-on-sub-classification-of-sc-sts- needs-to-be-reconsidered-6572630/ & Indian The second provision of Article 15 says that no Polity – M Laxmikanth, Chapter – 7 citizen shall be subjected to any disability, ‘Fundamental Rights’. liability, restriction or condition on grounds only of religion, race, caste, sex, or place of birth with Q.30) Which of the following is/are the most regard to (a) access to shops, public restaurants, likely result of the appreciation of Indian rupee hotels and places of public entertainment; or (b) against the US dollar? the use of wells, tanks, bathing ghats, road and 1. Indian exports become more competitive. places of public resort maintained wholly or 2. Indian imports become cheaper. partly by State funds or dedicated to the use of 3. Foreign travel and education become cheaper. general public. This provision prohibits Select the correct answer using the code given discrimination both by the State and private below. individuals, while the former provision prohibits a) 1 only discrimination only by the State. b) 2 and 3 only

c) 1 and 3 only 20investor%20sentiment. & d) 1, 2 and 3 https://www.livemint.com/Money/QRl7U8L7T8 cIneTwTwDw6J/A-stronger-rupee-who-wins- Ans) b who-loses.html & Exp) Currency appreciation is an increase in https://www.investopedia.com/terms/c/currency- the value of one currency in relation to another appreciation.asp & currency. Currencies appreciate against each https://www.financialexpress.com/money/how- other for a variety of reasons, including does-the-depreciating-rupee-impact-your- government policy, interest rates, trade balances finances/1290532/ and business cycles. Q.31) Consider the following statements: Effects of Currency Appreciation 1. Dengue disease is transmitted by the same When a nation's currency appreciates, it can have mosquito that transmits Zika and a number of different effects on the economy. Chikungunya viruses. • Export costs rise: If the Indian rupee 2. Maternal transmission of Dengue disease is appreciates, foreigners will find Indian goods possible. more expensive because they have to spend Which of the statements given above is/are more for those goods in USD. That means correct? that with the higher price, the number of a) 1 only Indian goods being exported will likely drop. b) 2 only Thus appreciation of Indian rupee leads to c) Both 1 and 2 Indian exports becoming less competitive. d) Neither 1 nor 2 Hence, statement 1 is incorrect. • Cheaper imports: If Indian rupee Ans) c appreciates, it means one can buy more goods Exp) Dengue - Dengue is a mosquito-borne in the international market with a lesser viral disease that has rapidly spread in all regions amount of rupee than required previously. in recent years. Dengue virus is transmitted by Thus Indian imports become cheaper. female mosquitoes mainly of the species Aedes Hence, statement 2 is correct. aegypti and, to a lesser extent, Ae. albopictus. • Foreign travel and education become These mosquitoes are also vectors of cheaper: When the rupee appreciates, it chikungunya, yellow fever and Zika viruses. gains value with respect to the dollar. This Dengue is widespread throughout the tropics, means it takes less rupees to exchange with a with local variations in risk influenced by rainfall, dollar. Thus foreign travel and education temperature, relative humidity and unplanned become cheaper. Hence, statement 3 is rapid urbanization. Hence, statement 1 is correct. correct.

Source) The primary mode of transmission of Dengue https://indianexpress.com/article/business/marke virus between humans involves mosquito vectors. t/rupee-gains-48p-against-dollar-on-steady- There is evidence however, of the possibility of foreign-fund-inflows-rbi-guv-comments- maternal transmission (from a pregnant 6572708/#:~:text=The%20rupee%20on%20Thu mother to her baby). While vertical rsday%20gained,currency%20boosted%20the%

transmission rates appear low, with the risk of securities — with original maturity of one year vertical transmission seemingly linked to the or more. timing of the dengue infection during the pregnancy. When a mother does have a Dengue In India, the central government issues both: virus infection when she is pregnant, babies may treasury bills and bonds or dated securities, while suffer from preterm birth, low birthweight, and state governments issue only bonds or dated fetal distress. Hence, statement 2 is correct. securities, which are called the state development loans. Since they are issued by the Source) government, they carry no risk of default, and https://indianexpress.com/article/explained/mosq hence, are called risk-free gilt-edged instruments. uitoes-dengue-indonesia-experiment-6572699/ & Hence, statement 1 is correct but statement 2 https://www.cdc.gov/dengue/transmission/index. is incorrect. html & https://www.who.int/news-room/fact- sheets/detail/dengue-and-severe-dengue Source) https://www.thehindubusinessline.com/money- Q.32) With reference to the Government and-banking/rbis-20000-crore-special-omo-has- Securities (G-Secs), consider the following only-temporary-cooling-effect-on-g-sec- statements: yields/article32448875.ece & 1. G-Secs with maturity of less than one year are https://economictimes.indiatimes.com/markets/b called treasury bills while those with maturity onds/what-are-govt-securities-and-how-to-buy- of one year or more are called government them/articleshow/67070971.cms bonds. 2. In India, Government Securities can only be Q.33) With reference to the ‘Question Hour’, issued by the Central government. consider the following statements: Which of the statements given above is/are 1. A starred question requires an oral answer correct? whereas an unstarred question requires a a) 1 only written answer. b) 2 only 2. During question hour, questions can also be c) Both 1 and 2 asked to the private members. d) Neither 1 nor 2 Which of the statements given above is/are correct? Ans) a a) 1 only Exp) What is a Government Security (G-Sec)? b) 2 only A government security (G-Sec) is a tradable c) Both 1 and 2 instrument issued by the central government d) Neither 1 nor 2 or state governments. It acknowledges the government’s debt obligations. Such securities Ans) c are short term — called treasury bills — with Exp) Question Hour original maturities of less than one year, or The first hour of every parliamentary sitting is long term — called government bonds or dated slotted for this. During this time, the members ask questions and the ministers usually give

answers. The questions are of three kinds, 2. All deposits under small savings schemes are namely, starred, unstarred and short notice. credited to this fund. • A starred question (distinguished by an 3. The fund is administered independently of asterisk) requires an oral answer and hence the government by the Reserve Bank of supplementary questions can follow. India. • An unstarred question, on the other hand, Select the correct answer using the code given requires a written answer and hence, below. supplementary questions cannot follow. a) 1 and 2 only (Hence, statement 1 is correct.) b) 2 and 3 only • A short notice question is one that is asked c) 1 and 3 only by giving a notice of less than ten days. It is d) 1, 2 and 3 answered orally. • In addition to the ministers, the questions Ans) a can also be asked to the private members. Exp) National Small Savings Fund (NSSF) Thus, a question may be addressed to a Small Saving schemes have always been an private member if the subject matter of the important source of household savings in India. question relates to some Bill, resolution or Small savings instruments can be classified under other matter connected with the business of three heads. These are: (i) postal deposits the House for which that member is [comprising savings account, recurring deposits, responsible. The procedure in regard to such time deposits of varying maturities and monthly a question is the same as that followed in the income scheme(MIS)]; (ii) savings certificates case of questions addressed to a minister. [(National Small Savings Certificate VIII (NSC) (Hence, statement 2 is correct.) and Kisan Vikas Patra (KVP)]; and (iii) social security schemes [(public provident fund (PPF) The list of starred, unstarred, short notice and Senior Citizens‘ Savings Scheme(SCSS)]. questions and questions to private members are printed in green, white, light pink and yellow A National Small Savings Fund (NSSF) in the colour, respectively, to distinguish them from one Public Account of India has been established another. with effect from 1.4.1999. A new sub sector has been introduced called “National Small Savings Source) Fund” in the list of Major and Minor Heads of https://indianexpress.com/article/india/dont- Government Accounts. All small savings curtail-question-hour-congress-lok-sabha-leader- collections are credited to this Fund. Similarly, adhir-tells-speaker-6574221/ & all withdrawals under small savings schemes by http://164.100.47.194/loksabha/FAQ.aspx & the depositors are made out of the accumulations Indian Polity – M Laxmikanth, Chapter – 22 in this Fund. The balance in the Fund is invested ‘Parliament’. in Central and State Government Securities. The investment pattern is as per norms decided from Q.34) Which of the following statements is/are time to time by the Government of India. (Hence, correct regarding ‘National Small Savings Fund statements 1 and 2 are correct.) (NSSF)’? 1. It is a fund established under the Public Account of India.

The Fund is administered by the Government b) 2 only of India, Ministry of Finance (Department of c) Both 1 and 2 Economic Affairs) under National Small d) Neither 1 nor 2 Savings Fund (Custody and Investment) Rules, 2001, framed by the President under Ans) d Article 283(1) of the Constitution. (Hence, Exp) While undertrials in Indian prisons statement 3 is incorrect.) increased from 67 per cent in 2015 to 69 per cent in 2019, capacity in jails increased by 1.9 The objective of NSSF is to de-link small per cent during this period, according to latest savings transactions from the Consolidated data on jails released by the National Crime Fund of India and ensure their operation in a Records Bureau (NCRB). (Statements 1 is transparent and self-sustaining manner. Since incorrect.) NSSF operates in the public account, its transactions do not impact the fiscal deficit of the More than 18 lakh people were brought to jails in Centre directly. As an instrument in the public 2019, of which over 3 lakh continue to languish account, the balances under NSSF are direct without any recourse to bail, the data for 2019 liabilities and constitute a part of the shows. outstanding liabilities of the Centre. The NSSF flows affect the cash position of the Central It also points to overcrowding in prisons – Government. from an occupancy rate of 114 per cent in 2015 to 118.5 per cent in 2019. (Statements 2 is Source) incorrect.) https://indianexpress.com/article/business/econo my/citing-strained-finances-states-ask-centre-to- The highest occupancy rate was reported from borrow-6574124/ & jails in Delhi, where almost 175 prisoners are http://www.arthapedia.in/index.php?title=Nation crammed in a space meant for 100. Uttar Pradesh al_Small_Savings_Fund & (167.9 per cent) and Uttarakhand (159 per cent) http://www.nsiindia.gov.in/InternalPage.aspx?Id follow Delhi, NCRB reported. _Pk=54 The educational profile of jail inmates (including undertrials and convicts) shows they are Q.35) With reference to the recently released predominantly illiterate or semi-literate. While data on prisons by the National Crime Records 27.7 percent prisoners were illiterate, 41.6 Bureau, consider the following statements: percent prisoners had education below Class 1. The number of undertrials as a percentage of X, 21.5 per cent had passed Class X but were total prisoners have decreased steadily in the below graduation. In effect, almost 70 per cent last 5 years. inmates are either illiterate or semi-literate and 2. The occupancy rate of prisons have more than 90 per cent had not gone to college, decreased steadily in the last 5 years. NCRB data shows. Only 6.3 per cent prisoners Which of the statements given above is/are had a degree, and 1.7 per cent were postgraduates. correct? a) 1 only

Source) electricity, registering property, getting credit, https://indianexpress.com/article/india/at-118- protecting minority investors, paying taxes, inmates-in-space-for-100-share-of-undertrials- trading across borders, enforcing contracts, and in-jails-up-ncrb-data-6575614/ & resolving insolvency. https://indianexpress.com/article/india/ncrb- data-higher-share-of-dalits-tribals-muslims-in- An 11th area – employing workers – is measured, prison-than-numbers-outside-6575446/ but not factored into the score.

Q.36) Recently the World Bank says it is pausing Option c is incorrect. Sustainable economic publishing of the Doing Business 2021 report and development is not an indicator covered under will conduct an internal audit of reports for the the report. past 5 years. Which of the following is not an indicator covered under this report? The Doing Business report presents results for a) Protection of minority investors two aggregate measures: the distance to b) Trading across borders frontier score and the ease of doing business c) Sustainable economic development ranking, which is based on the distance to d) Dealing with Construction permit frontier score. The ease of doing business ranking compares economies with one another; the Ans) c distance to frontier score benchmarks economies Exp) Recently the World Bank has decided to with respect to regulatory best practice, showing pause the publication of its ‘Doing Business the absolute distance to the best performance on 2021’ report in view of the fact that data from each Doing Business indicator. Azerbaijan, China, Saudi Arabia and the United Distance to Frontier (DTF) score highlights the Arab Emirates were found to have been gap of an economy with respect to global best inappropriately altered in the 2018 and 2020 practice. For example, a score of 75 means an reports. economy was 25 percentage points away from the India ranks 63rd among 190 countries improving frontier constructed from the best performances by 14 ranks from its rank of 77 in 2019 in World across all economies and across time. Bank’s Ease of Doing Business ranking 2020.

Doing Business’ is the World Bank’s flagship annual report on business competitiveness. They measure regulations that enhance business activity and those that constrain it. The reports carry the ease of doing business rankings.

The indicator measures the performance of countries across 10 different dimensions in the 12-month period ending May 1, 2019.

The 10 areas of study are: starting a business, dealing with construction permits, getting

Source) https://theprint.in/economy/data-of-4- varying degrees of autonomy within the state nations-manipulated-in-2018-2020-doing- legislature. business-reports-world-bank-to-conduct- audit/490964/ The governors of these states are empowered to https://www.jatinverma.org/indias-ease-of- reorganise boundaries of the tribal areas. In doing-business-ranking-2020 simpler terms, she or he can choose to include or https://www.thehindu.com/business/india- exclude any area, increase or decrease the climbs-14-notches-in-ease-of-doing-business- boundaries and unite two or more autonomous ranking/article29789723.ece districts into one. They can also alter or change the names of autonomous regions without a Q.37) With reference to Indian Polity, consider separate legislation. the following statements: 1. A Legislative assembly cannot pass Acts passed by Parliament and state resolution for inclusion of a state in the sixth legislatures may or may not be levied in these schedule of the Indian Constitution. regions unless the President and the governor 2. An act of Parliament can never be applied in gives her or his approval, with or without tribal areas covered under the sixth schedule. modifications in the laws for the autonomous Which of the statements given above is/are regions. correct? a) 1 only Source) https://theprint.in/theprint- b) 2 only essential/what-is-6th-schedule-why-it-allows- c) Both 1 and 2 parts-of-northeast-to-be-exempt-from- d) Neither 1 nor 2 citizenship-bill/331404/ https://economictimes.indiatimes.com/news/p Ans) d olitics-and-nation/arunachal-assembly- Exp) Both statements are incorrect. passes-resolution-for-inclusion-of-state-in- Recently Arunachal Pradesh Legislative sixth-schedule/articleshow/77789660.cms assembly unanimously passed a resolution for the entire state to be included in the Sixth Q.38) The term “Hangenberg crisis” recently Schedule of the constitution. seen in the news is related to a) loss of species diversity due to depletion of Sixth Schedule ozone. The Sixth Schedule consists of provisions for the b) the formation of black holes. administration of tribal areas in Assam, c) the uncertainty in behaviors of tiny objects Meghalaya, Tripura and Mizoram, according to like photons. Article 244 of the Indian Constitution. d) disappearance of good native crop plants and the loss of food biodiversity due to large scale Passed by the Constituent Assembly in 1949, it cultivation of genetically modified crops seeks to safeguard the rights of tribal population through the formation of Autonomous District Ans) a Councils (ADC). ADCs are bodies representing a Exp) district to which the Constitution has given Explosion of a nearby star — occurred at between Devonian and Carboniferous periods — could

have caused a mass extinction event that took autotrophic and heterotrophic, respectively. place 359 million years ago. Algae prepare food for fungi and fungi provide The Earth suffered an intense loss of species shelter and absorb mineral nutrients and water for diversity that lasted for at least 300,000 years. its partner. So close is their association that if one The event is thought to have been caused by saw a lichen in nature one would never imagine long-lasting ozone depletion, which would have that they had two different organisms within allowed much more of the Sun’s ultraviolet (UV) them. Lichens are very good pollution radiation to reach and harm life on Earth. It was indicators – they do not grow in polluted areas. called the Hangenberg crisis. Statement 2 is incorrect. Source) Mutualism: This interaction confers benefits on https://www.downtoearth.org.in/news/science- both the interacting species. Lichens represent an technology/did-a-supernova-trigger-the-late- intimate mutualistic relationship between a devonian-extinction--73088 fungus and photosynthesising algae or cyanobacteria. Similarly, the mycorrhizae are Q.39) Consider the following statements associations between fungi and the roots of regarding Lichens: higher plants. The fungi help the plant in the 1. Lichens are the symbiotic associations absorption of essential nutrients from the soil between algae and fungi. while the plant in turn provides the fungi with 2. Lichens exhibit the commensalism type of energy-yielding carbohydrates. population interactions. Commensalism: This is the interaction in which 3. Lichens are used as pollution indicators. one species benefits and the other is neither harmed nor benefited. An orchid growing as an Which of the statements given above is/are epiphyte on a mango branch, and barnacles correct: growing on the back of a whale benefit while a) 1 only neither the mango tree nor the whale derives any b) 2 and 3 only apparent benefit c) 1 and 3 only d) 1, 2 and 3 Interspecific interactions arise from the interaction of populations of two different Ans) c species. They could be beneficial, detrimental or Exp) Recently Uttarakhand forest department has neutral (neither harm nor benefit) to one of the developed the country’s first lichen park in species or both. Assigning a ‘+’ sign for Munsiyari, Uttarakhand. The park has been beneficial interaction, ‘-’ sign for detrimental and developed with an aim to conserve, protect, and 0 for neutral interaction, let us see at all the cultivate lichens and to create awareness among possible outcomes of interspecific interactions. locals regarding their importance.

Statement 1 and 3 are correct. Lichens are symbiotic associations i.e. mutually useful associations, between algae and fungi. The algal component is known as phycobiont and fungal component as mycobiont, which are

respective states to the secretary, Union Ministry of Environment, Forest and Climate Change (MoEF&CC) within three months.

This order of NGT, was in response to the application on the unscientific dumping of waste and encroachment of the Hokersar wetland, Wular lake and Kreentchoo-Chandhara wetland in the Union Territory of Jammu & Kashmir. Hence, option 3 is correct.

Source) Source) https://www.downtoearth.org.in/news/govern https://timesofindia.indiatimes.com/city/dehradu ance/daily-court-digest-major-environment- n/ukhand-forest-department-develops-indias- orders-august-31-2020--73147 first-lichen-park-in- munsyari/articleshow/76666225.cms’ https://ncert.nic.in/textbook/pdf/kebo102.pdf Q.41) Consider the following https://ncert.nic.in/textbook/pdf/kebo102.pdf 1. Gangetic River Dolphin 2. Golden Mahseer Q.40) Consider the following pairs: 3. Asiatic Lion Wetlands State/Union Territory Which of the above species is/are listed as 1. Hokersar Telangana endangered in the IUCN red list? 2. Kreentchoo-Chandhara Manipur a) 1 and 2 only 3. Wular Lake Jammu & Kashmir b) 2 and 3 only Which of the pairs given above is/are correctly c) 3 only matched? d) 1, 2 and 3 a) 1 and 3 only b) 2 and 3 only Ans) d c) 3 only Exp) All options are correct. d) 1, 2 and 3 Golden Mahseer - Golden Mahseer lives in fast-moving waters, Ans) c inhabiting hill streams with a rocky and stony Exp) Options 1 and 2 are incorrect. substrate. They can be found in temperatures Recently the National Green Tribunal (NGT) between 5°C and 25°C. The fish has also been directed the National Wetlands Committee to introduced in lakes and occurs in large reservoirs. compile data on status of compliance of The Golden Mahseer inhabits the Himalayan environmental norms in all significant wetlands foothills, the Indus, Ganga and Brahmaputra of India to ensure remedial action. basins and can also be found down south in the The state pollution control boards (SPCB) / Balamore, Cauvery, Tambraparini, and Kosi pollution control committees (PCC) and state / Rivers. Upon maturity, the adults inhabit lowland union territory (UT) wetland authorities have to rivers and lakes and migrate upstream in give the status of wetland management in their

torrential monsoon conditions to reach suitable https://www.wwfindia.org/about_wwf/priority_s spawning grounds. It has been listed as pecies/threatened_species/ganges_river_dolphin/ Endangered in IUCN red list. https://www.wwfindia.org/about_wwf/priority_s pecies/threatened_species/asiatic_lion/ Asiatic Lion. wwfindia.org/about_wwf/priority_species/threat Asiatic lions were once distributed upto the state ened_species/golden_mahseer/ of West Bengal in east and Rewa in Madhya Pradesh, in central India. At present Gir National Q.42) The term Black Rock recently seen in the Park and Wildlife Sanctuary is the only abode of news is a/an the Asiatic lion. The last surviving population of a) android malware equipped with data stealing the Asiatic lions is a compact tract of dry capacity. deciduous forest and open grassy scrublands in b) stealth fifth generation jet fighter of USA southwestern part of Saurashtra region of c) asteroid found in venus belt Gujarat.It has been listed as Endangered in d) merchandise based cryptocurrency IUCN red list. Ans) a Gangetic river Dolphin Exp) Ganges river dolphins prefer deep waters, in and The country’s cyber security agency(CERT-In) around the confluence of rivers. The distribution has issued an alert against an Android malware, range of the Ganges river dolphins in India covers dubbed ‘BlackRock’, that has the potential to seven states namely, Assam, Uttar Pradesh, “steal” banking and other confidential data of a Madhya Pradesh, Rajasthan, Bihar, Jharkhand user. It can extract credentials and credit card and West Bengal. The upper Ganga River (in information from over 300 apps such as email, e- Uttar Pradesh), Chambal River (Madhya Pradesh commerce apps, social media apps, besides and Uttar Pradesh), Ghaghra and Gandak Rivers banking- and financial apps. (Bihar and Uttar Pradesh), Ganga River, from Varanasi to Patna (Uttar Pradesh and Bihar), Son It was recently reported that a new Android and Kosi rivers (Bihar), Brahmaputra from Sadia malware strain dubbed ‘BlackRock’ equipped (foothills of Arunachal Pradesh) upto Dhubri (on with data-stealing capabilities is attacking a wide the Bangladesh border) and Kulsi River, a range of Android applications. tributary of the Brahmaputra River, form ideal habitats for the Ganges river dolphin. It has been The malware is developed using the source code listed as Endangered in IUCN red list. of Xerxes banking malware which itself is a variant of LokiBot Android Trojan. The Source) “noteworthy feature” of this malware is that its https://theprint.in/environment/after-project- target list contains 337 applications including tiger-and-project-elephant-india-to-have-project- banking and financial applications and also non- dolphin-says-pm-modi/482450/ financial and well-known commonly used brand https://www.thehindubusinessline.com/blink/cov name apps on an Android device that focus on er/clipped-choked-and-cordoned- social, communication, networking and dating off/article32351647.ece platforms

Source) https://www.thehindu.com/sci- These notes allow foreign high networth tech/technology/internet/android-malware- individuals, hedge funds and other investors to blackrock-prowling-in-cyber-space-may-steal- put money in Indian markets without being banking-data-cert-in/article32231034.ece registered with SEBI, thus making their participation easy and smooth. P-Notes also aid Q.43) With reference to Participatory notes (P- in saving time and costs associated with direct notes), consider the following statements: registrations. Hence, statement 2 is incorrect. 1. P-Notes are offshore derivative instruments. 2. P- Notes can only be used by Foreign Source) Institutional Investors registered with the https://economictimes.indiatimes.com/markets/st SEBI to invest in Indian securities. ocks/news/p-notes-investment-continues-to- Which of the statements given above is/are soar-climbs-to-over-rs-63000-crore-till-jul- correct? end/articleshow/77675598.cms a) 1 only https://www.indiainfoline.com/article/news- b) 2 only sector-others/what-are-participatory-notes- c) Both 1 and 2 113111501088_1.html d) Neither 1 nor 2 Q.44) With reference to the Banks Board Bureau Ans) a sometimes seen in the news, consider the Exp) Statement 1 is correct. following statements: Participatory notes also called P-Notes are 1. It is an autonomous body. offshore derivative instruments with Indian 2. Its main function is to improve corporate shares as underlying assets. These instruments governance at public sector banks. are used for making investments in the stock 3. It was formed on the recommendation of PJ markets. However, they are not used within the Nayak committee. country. They are used outside India for making Which of the statements given above is/are investments in shares listed in the Indian stock correct? market. That is why they are also called offshore a) 1 and 2 only derivative instruments. b) 2 and 3 only c) 1 and 3 only Participatory notes are issued by brokers and FIIs d) 1, 2 and 3 registered with SEBI. The investment is made on behalf of these foreign investors by the already Ans) d registered brokers in India. For example, Indian- Exp) All statements are correct. based brokerages buy India-based securities and Banks Board Bureau has its genesis in the then issue participatory notes to foreign investors. recommendations of ‘The Committee to Any dividends or capital gains collected from the Review Governance of Boards of Banks in underlying securities go back to the investors. India, May 2014 (P. J. Nayak committee)’. It is an autonomous body. The brokers that issue these notes or trades in Indian securities have to mandatorily report their The government, in 2016, approved the PN issuance status to SEBI for each quarter. constitution of the BBB as a body of eminent

professionals and officials to make Exp) Both statements are correct. recommendations for appointment of whole-time National Highways Authority of India was set directors as well as non-executive chairpersons of up by an act of the Parliament, NHAI Act, Public Sector Banks (PSBs) and state-owned 1988(Hence it is a statutory body). It has been financial institutions. entrusted with the National Highways However, the Ministry of Finance takes the final Development Project, which along with other decision on the appointments in consultation with minor projects, has vested in its 50329 kms of the Prime Minister’s Office. National Highways for development, ● It has also been assigned with the task of maintenance and management. recommending personnel for appointment as directors in government-owned insurance NHAI objective is to ensure that all contract companies. awards and procurements conform to the best ● BBB engages with the board of directors of industry practices with regard to transparency of all the public sector banks to formulate process, adoption of bid criteria to ensure healthy appropriate strategies for their growth and competition in award of contracts, development. implementation of projects conform to best ● It is also tasked with improving corporate quality requirements and the highway system is governance at public sector banks, maintained to ensure best user comfort and building capacities, etc. convenience.

Source) Recently according to an NHAI release, to https://www.financialexpress.com/industry/bank commemorate 25 years of its service to the ing-finance/hunt-for-sidbi-deputy-md-banks- nation, it has also recently undertaken ‘Harit board-bureau-seeks-applications-for-psu-bank- Bharat Sankalp’, a nation-wide plantation drive post/2057842/ which is in line with its commitment to promote https://www.jatinverma.org/reserve-bank- environment protection and sustainability. Under proposes-to-shore-up-bank-boards this initiative, NHAI planted over 25 lakh plants in 25 days along the stretches of the National Q.45) Which of the following statements is/are Highways between 21st July to 15th August correct regarding National Highway Authority of 2020. The drive takes the total cumulative India(NHAI)? number of plantations done during the current 1. It is a statutory body. year to 35.22 lakh. 2. Harit Bharat Sankalp initiative is a nationwide plantation drive of NHAI along Source) the stretches of the National Highways. https://pib.gov.in/PressReleseDetailm.aspx?PRI Select the answer using the code given below. D=1647625 a) 1 only https://nhai.gov.in/about-nhai.htm b) 2 only c) Both 1 and 2 Q.46) Consider the following: d) Neither 1 nor 2 1. Salween 2. Sutlej Ans) c 3. Gandak

4. Kosi the Indus in the mid-section of Pakistan, joining 5. Jhelum forces for the final rush to the sea. Which of the above rivers do not originate from India? Salween river flows over 2,800 kilometres from a) 1, 2, 3 and 4 only source to sea. It rises in the Tangula Mountain b) 3, 4 and 5 only region (Amdo, Tibetan plateau) and courses c) 1, 3 and 4 only through Tibet, southwest China and Burma d) All of the above (shared with the Thai border), before finally emptying into the Andaman Sea. Ans) a Exp) The Koshi or drains the northern Rivers given in options 1, 2 , 3 and 4 do not slopes of the in the Tibet Autonomous originate in India. As can be seen in the map Region and the southern slopes in . From a below: major confluence of tributaries north of the Chatra Gorge onwards, the Koshi River is also Option 5 is correct. known as Saptakoshi for its seven upper The Jhelum River is a river in northern India and tributaries. These include the Tamur Koshi eastern Pakistan. It is the westernmost of the five originating from the Kanchenjunga area in the rivers of the Punjab region, and passes through east, Arun River from Tibet and Sun Koshi from the Kashmir Valley. It is a tributary of the Chenab the Gosainthan area farther west. The Sun Koshi's River and has a total length of about 725 tributaries from east to west are , kilometres. Bhote Koshi, Tamba Koshi and Indravati Koshi. The river Jhelum rises from Verinag Spring The Saptakoshi crosses into northern Bihar where situated at the foot of the Pir Panjal in the it branches into distributaries before joining the southeastern part of the Kashmir Valley. It flows Ganges near Kursela in Katihar district. through Srinagar and Wular Lake before entering Pakistan through a deep narrow gorge.

Gandak River, also called Narayani River, is a river in central Nepal and northern India. It is formed by the union of the Kali and Trisuli rivers, which rise in the Great Himalaya Range in Nepal; from this junction to the Indian border the river is called the Narayani. It flows southwest into India and then turns southeast along the Uttar Pradesh– Bihar state border and across the Indo-Gangetic Plain

Sourced from Tibet—the Mount Kailash region—the Sutlej River (Langchan Khambab) carves a quite different path, but meets up with

Ans) c Exp) Both statements are correct. POSHAN Abhiyaan or National Nutrition Mission, is the Government of India’s flagship programme to improve nutritional outcomes for children, pregnant women and lactating mothers.

Launched by the Prime Minister on the occasion of the International Women’s Day on 8 March, 2018 from Jhunjhunu in Rajasthan, the POSHAN (Prime Minister’s Overarching Scheme for Holistic Nutrition) Abhiyaan directs the attention of the country towards the problem

of malnutrition and address it in a mission-mode.

Source) POSHAN Abhiyaan (National Nutrition https://www.mapsofindia.com/maps/india/india- Mission) is a flagship programme of the Ministry river-map.htm of Women and Child Development (MWCD), https://docs.google.com/document/d/1f1QjyMY Government of India, which ensures JUHAAt1LOZ7H9I6rIXKLVrJqzCmUS7ovm1 convergence with various programmes i.e., 8w/edit Anganwadi Services, Pradhan Mantri Matru https://en.wikipedia.org/wiki/Jhelum_River Vandana Yojana (PMMVY), Scheme for http://117.252.14.242/Gangakosh/tributaries/kos Adolescent Girls (SAG) of MWCD Janani i.htm Suraksha Yojana (JSY), National Health Mission https://www.google.com/url?sa=i&url=https%3 (NHM), Swachh-Bharat Mission, Public A%2F%2Fwww.meltdownintibet.com Distribution System (PDS), Department Food &

Public Distribution, Mahatma Gandhi National Q.47) With reference to Poshan Abhiyaan Rural Employment Guarantee Scheme sometimes seen in the news, consider the (MGNREGS) and Ministry of Drinking Water & following statements: Sanitation. 1. It aims to improve nutritional outcomes for NITI Aayog has played a critical role in shaping pregnant women and lactating mothers. the POSHAN Abhiyaan. The National Nutrition 2. It will ensure convergence of Pradhan Mantri Strategy, released by NITI Aayog in September, Matru Vandana Yojana, Swachh Bharat 2017 presented a micro analysis of the problems Mission and Public Distribution System. persisting within this area and chalked out an in- Which of the statements given above is/are depth strategy for course correction. Most of the correct? recommendations presented in the Strategy a) 1 only document have been subsumed within the design b) 2 only of the POSHAN Abhiyaan and now that the c) Both1 and 2 Abhiyaan is launched, NITI Aayog has been d) Neither 1 nor 2 entrusted with the task of closely monitoring the

POSHAN Abhiyaan and undertaking periodic Which of the statements given above is/are evaluations. correct? a) 1 and 2 only With the overarching aim to build a people’s b) 2 and 3 only movement (Jan Andolan) around malnutrition, c) 1 and 3 only POSHAN Abhiyaan intends to significantly d) 1, 2 and 3 reduce malnutrition in the next three years. For implementation of POSHAN Abhiyaan the Ans) c four point strategy/pillars of the mission are: Exp) Statements 1 and 3 are correct. ● Inter-sectoral convergence for better service WWF(World Wide Fund for Nature)-India State delivery unit along with the Society for Odonate Studies ● Use of technology (ICT) for real time growth (SOS) for the first-ever State Dragonfly Festival monitoring and tracking of women and in Kerala, christened Thumbi Mahotsavam 2020. children This is part of a national dragonfly festival being ● Intensified health and nutrition services for organised by the WWF India, Bombay Natural the first 1000 days History Society & Indian Dragonfly Society in ● Jan Andolan association with the National Biodiversity Board, United Nations Environment NNM targets to reduce stunting, under- nutrition, Programme, United Nations Development anemia (among young children, women and Programme and IUCN - Centre for adolescent girls) and reduce low birth weight by Environment Conservation. 2%, 2%, 3% and 2% per annum respectively. Although the target to reduce Stunting is at least Statement 2 is incorrect. Pantalu is the official 2% p.a., Mission would strive to achieve mascot of the festival. reduction in Stunting from 38.4% (NFHS-4) to (Budha Mayoori is the State butterfly of Kerala) 25% by 2022 (Mission 25 by 2022). About Dragonflies Source) ● The Odonata are an order of flying insects https://www.financialexpress.com/opinion/covid that includes the dragonflies and damselflies -19-feeding-the-future-of-india/2063264/ which are critical bioindicators. https://niti.gov.in/poshan-abhiyaan ● Dragonflies are some of the best predators to https://vikaspedia.in/health/nrhm/national- keep mosquito populations low as they health-programmes-1/national-nutrition-mission scavenge the skies in adulthood, and also eat

a large number of mosquito larvae. Q.48) Consider the following statements regarding Thumbi Mahotsavam: Source) 1. It is the first-ever State Dragonfly Festival in https://www.thehindu.com/news/national/kerala/ Kerala. kerala-to-host-dragonfly- 2. Budha Mayoori is the official mascot of the festival/article32414553.ece festival. https://www.jatinverma.org/kerala-to-host- 3. The festival is being organised by Bombay dragonfly-festival Natural History Society.

Q.49) “This organisation was founded in 1961 About IUCN(International Union for and is the world’s largest conservation Conservation of Nature): organization. It is an international non- ● IUCN is a membership union uniquely governmental organization for conserving the composed of both government and civil world’s biological diversity and it publishes society organisations. It was created in 1948, ‘Living Planet Report’ every two years which is it is the global authority on the status of the based on a Living Planet Index and ecological natural world and the measures needed to footprint calculation.” safeguard it. The organisation mentioned in the above ● Headquarter is in Switzerland. paragraph is ● The IUCN Red List of Threatened Species, a) International Union for Conservation of Nature is the world's most comprehensive inventory b) World Wide Fund for Nature of the global conservation status of plant and c) United Nations Environment Programme animal species. d) Bombay Natural History Society About UNEP Ans) b ● The UNEP is an international environmental Exp) About WWF (World Wide Fund for authority engaged in establishing a global Nature): environmental agenda and promoting the ● It is an international non-governmental efficient implementation of the organization for conserving the world’s environmental dimension of the United biological diversity. It was founded in 1961 Nations Sustainable Development and is the world’s largest conservation Programme. organization. ● It was founded as a result of Stockholm ● Headquarter — Gland (Switzerland). conference (Un Conference on Human ● WWF publishes ‘Living Planet Report’ Environment) in 1972. every two years by WWFwhich is based on ● Headquarter is in Nairobi , kenya a Living Planet Index and ecological footprint calculation. Source) https://www.newindianexpress.com/states/kerala About BNHS: /2020/aug/08/with-28-bird-hits-last-year- ● The Bombay Natural History Society is a experts-say-flights-to-thiruvananthapuram-also- non-governmental organisation in India, fraught-with-risk-2180849.html founded in 1883. https://www.jatinverma.org/kerala-to-host- dragonfly-festival ● It is engaged in conservation and biodiversity

research. Q.50) With reference to Intercreditor agreement ● BNHS is the partner of BirdLife International sometimes seen in the news, consider the in India. following statements: ● It has been selected as a ‘Scientific and 1. It aims to lessen the Government of India’s Industrial Research Organisation’ by the perennial burden of fiscal deficit. Department of Science and Technology.

2. It is a part of Sashakt plan of government of c) Ministry of Culture is the nodal ministry for India to address the problem of resolving bad coordination of the programme. loans. d) Exchange of information on traditional Which of the statements given above is/are not agricultural practices is one of the key activities correct? under the scheme. a) 1 only b) 2 only Ans) c c) Both1 and 2 Exp) Option c is incorrect. “Ek Bharat d) Neither 1 nor 2 Shreshtha Bharat” was announced by Hon’ble Prime Minister on 31st October, 2015 on the Ans) a occasion of the 140th birth anniversary of Sardar Exp) Statement 1 is incorrect but statement 2 Vallabhbhai Patel.The Ministry of Human is correct. Resource Development has been designated Intercreditor Agreement is the direct result of Nodal Ministry for coordination of the the government's Sashakt resolution plan or programme. report on bad banks drafted by the Sunil Mehta committee. (It is related to resolution of All States and UTs will be covered under the NPA of banks not related to fiscal deficit). programme. There will be pairing of States/UTs at national level and these pairings will be in The lenders in the agreement will jointly appoint effect for one year, or till the next round of a lead lender who will function on behalf of the pairings. The State/UT level pairings would be entire group. The leading lender will then be utilized for state level activities. District level required to put forth the resolution plan for the pairings would be independent of the State level non-performing assets (NPAs) before the group pairings. and if it is given a go-ahead by two thirds of the The broad objectives of the initiative are as lenders, the proposal will qualify to be taken up follows:- for resolution of the given account. ● To CELEBRATE the Unity in Diversity of our Nation and to maintain and strengthen the Source) fabric of traditionally existing emotional https://indianexpress.com/article/explained/expla bonds between the people of our Country. ined-rbis-new-loan-recast-scheme-6544994/ ● PROMOTE the spirit of national https://www.jatinverma.org/rbi-sets-up-panel-to- integration through a deep and structured deal-with-coronavirus-related-stressed-assets engagement between all Indian States and Union Territories through a year-long planned engagement between States. Q.51) Which one of the following is not correct ● To SHOWCASE the rich heritage and regarding Ek Bharat Shreshtha Bharat Scheme? culture, customs and traditions of either State a) All States and Union Territories are covered for enabling people to understand and under this scheme. appreciate the diversity that is India, thus b) It will promote the spirit of national integration fostering a sense of common identity. between all Indian States and Union Territories

● TO CREATE an environment which 12. Organising NCC, NSS Camps of students of promotes learning between States by sharing one State at the locations in the partnering best practices and experiences. State.

Some key activities which have been Source) communicated to States for taking up suitably https://indiaeducationdiary.in/minister-sushri- under Ek Bharat Shreshtha Bharat are given thakur-to-inaugurate-ek-bharat-shreshtha-bharat- below: dekho-apna-north-east/ 1. Identification of proverbs having similar https://ekbharat.gov.in/Pages/AboutBackground meaning in the languages of the two States https://vikaspedia.in/social-welfare/social- and their translation and dissemination for awareness/schemes/ek-bharat-shreshtha-bharat interchangeable use. 2. Cultural Exchange Programmes between Q.52) With reference to Producer Responsibility partnering States with the help of troupes Organizations(PRO) model recently seen in the identified in the home State through Cultural news, consider the following statements: Competitions. 1. Under this model a producer can join a PRO 3. Educational Tour by University Students to fulfil their complete operational reciprocally in the partnering States to bring responsibility for acquiring plastic credits. out salient features of that State. 2. Municipal bodies can also be registered as 4. Exchange of information on traditional PRO. Agricultural Practices and Forecasting 3. The funding mechanism under this model among farmers in partnering States. will only be determined by the government 5. Organising Combined Tableau of Partnering on project to project basis. States on the Occasion of 15th August and Which of the statements given above is/are 26th January. correct? 6. Participation of Parade Contingents from one a) 1 and 2 only State in the Ceremonial Functions of the b) 3 only partnering State. c) 1 and 3 only 7. Organising National Quiz Competition in d) 1, 2 and 3 various languages on MyGov portal. 8. Taking lead in organising State Specific Quiz Ans) a Competition on TV / Radio / MyGov portal Exp) Statement 1 and 2 are correct. in the language of the partnering State. Recently Framework to make producers manage 9. Organising Photography competition for the plastic waste was released. The framework people of One State creating their portfolio suggests three different models to be adopted by on sites, places and objects in the Partnering manufacturers, importers and brand owners to State. handle the waste under supervision of a central 10. Organising Blog Competitions on Ek Bharat authority. Shreshtha Bharat. 11. Organising Cycling expeditions for the Introduction of a system of plastic credit, students / people of one State to the places in establishing Producer Responsibility the partnering State. Organisations (PROs) and setting up a fee-

based mechanism are three different models meet their targeted collection. This money will be which will be available to producers of plastic used for creating infrastructure for plastic waste products, including FMCG companies. management A single national registry will be created to enlist all stakeholders. It will improve A Producer can join a PRO to fulfil their monitoring and help bring transparency in the complete operational responsibility for plastic waste management system in the country acquiring plastic credits. PRO will carry out the where 40% of its daily waste remains legal requirements on behalf of their member uncollected, posing a threat to environmental and companies. However, the final responsibility of human health. providing final evidence of reprocessing/ export of plastic packaging rests with the producer. Under the ‘fee-based model’, the framework Based on the experience, PRO registration is proposed that the producers will contribute to the divided into 3-4 groups. EPR (Extended Producer EPR corpus fund at the central level. This may be Responsibility) targets will be decided upon the an escrow account, managed by SPV, where information provided by BO/ importers regarding private and other stakeholders can become the total number of states where they are members. The amount to be contributed by each operational, quality and quantity of plastics etc. of the producers will be calculated based on generation of plastic waste vis-a-vis efforts Under the PRO model, an organisation will required and money spent by urban local bodies manage the waste on behalf of producers. to handle the plastic part of the waste. Municipal bodies can also register as PRO or waste collector. There will be a national The draft framework says a plastic credit model authority - National PRO Advisory Committee - is envisaged where a producer is not required to to govern plastic waste management in the recycle their own packaging, but to ensure that an country. equivalent amount of packaging waste has been recovered and recycled to meet their obligation. Statement 3 is incorrect. PRO Funding Mechanism – Being a critical issue Source) for obligated producers in the design and https://timesofindia.indiatimes.com/india/frame operation of the PRO, it should be determined work-to-make-producers-manage-plastic-waste- through consultation with affected companies. is-out/articleshow/76635067.cms However, based on other global practises, per https://www.aipma.net/Environment/Summary- kilogram fee rate for each specific plastic is Uniform-EPR-Framework.pdf suggested, the methodology for determining the fee rates and the allocation should be determined Q.53) With reference to Global Partnership On by PRO, the per kg fee shall be calculated Artificial Intelligence(GPAI) recently seen in the considering all aspects. news, consider the following statements: 1. It is an international alliance under the The proposed uniform Extended Producers Financial Action Task Force to prevent Responsibility (EPR) framework, released by the global money laundering activities. Union environment ministry, also has provisions 2. India is a founding member of this alliance. to impose penalties on producers if they fail to

Which of the statements given above is/are international standards that aim to prevent these correct? illegal activities and the harm they cause to a) 1 only society. As a policy-making body, the FATF b) 2 only works to generate the necessary political will to c) Both 1 and 2 bring about national legislative and regulatory d) Neither 1 nor 2 reforms in these areas.

Ans) b The FATF reviews money laundering and Exp) Statement 1 is incorrect. terrorist financing techniques and continuously It is the first initiative of its type for evolving strengthens its standards to address new risks, better understanding of the challenges and such as the regulation of virtual assets, which opportunities around AI using the experience and have spread as cryptocurrencies gain popularity. diversity of participating countries. In order to The FATF monitors countries to ensure they achieve this goal, the initiative will look to bridge implement the FATF Standards fully and the gap between theory and practice on AI by effectively, and holds countries to account that do supporting cutting-edge research and applied not comply. activities on AI-related priorities(not an initiative under Financial Action Task Force) Source) https://www.livemint.com/news/india/india- GPAI will bring together experts from the becomes-founding-member-of-global- industry, civil society, governments, and partnership-for-artificial-intelligence- academia to collaborate to promote responsible 11592245966465.html evolution of AI and will also evolve https://pib.gov.in/PressReleasePage.aspx?PRID methodologies to show how it can be leveraged =1631676 to better respond to the present global crisis https://www.fatf- around Covid-19. gafi.org/about/#:~:text=The%20Financial%20A Statement 2 is correct. India joined GPAI as a ction%20Task%20Force,harm%20they%20caus founding member and will actively participate in e%20to%20society. the global development of Artificial Intelligence, leveraging its experience around use of digital Q.54) The term 2018VP1 , Didymos and Bennu technologies for inclusive growth. recently seen in the news are related to a) Asteroid Other members include the USA, UK, EU, b) Android based malware Australia, Canada, France, Germany, Italy, c) Earth-size planets exist in the habitable zones Japan, Mexico, New Zealand, Republic of Korea, of other stars Singapore. This body is supported by the d) Humanoid robot Organization for Economic Cooperation and Development (OECD), Paris. Ans) a Exp) Financial Action Task Force (FATF) is the Asteroid 2018VP1 is very small, approx. 6.5 global money laundering and terrorist financing feet, and poses no threat to Earth! It currently has watchdog. The inter-governmental body sets a 0.41% chance of entering our planet’s

atmosphere, but if it did, it would disintegrate due Which of the statements given above is/are to its extremely small size. correct? a) 1 and 2 only Didymos is a binary near-Earth asteroid, one b) 2 and 3 only of whose bodies is of the size that could pose the c) 1 and 3 only most likely significant threat to Earth. d) 1, 2 and 3 Asteroid Impact and Deflection Assessment (AIDA) includes NASA’s Double Asteroid Ans) b Redirection Test (DART) mission and the Exp) The countries across the globe committed European Space Agency’s (ESA) Hera and their to create a new international climate agreement target is Didymos, a binary near-Earth asteroid, by the conclusion of the U.N. Framework that could pose the most likely significant threat Convention on Climate Change (UNFCCC) to Earth. Conference of the Parties (COP21) in Paris in December 2015. In preparation, countries have Asteroid Bennu is the target of the OSIRIS-REx agreed to publicly outline what post-2020 climate mission which is intended to return samples to actions they intend to take under a new Earth in 2023 for further study. On 3 December international agreement, known as their Intended 2018, the OSIRIS-REx spacecraft arrived at Nationally Determined Contributions (INDCs). Bennu after a two-year journey. Before The INDCs will largely determine whether the attempting to obtain a sample from the asteroid, world achieves an ambitious 2015 agreement and it will map out Bennu's surface in detail and orbit is put on a path toward a low-carbon, climate- the asteroid to calculate its mass. resilient future.

Source) Salient features of India's INDC https://indianexpress.com/article/explained/what ● To put forward and further propagate a -is-asteroid-2018vp1-6568690/ healthy and sustainable way of living based https://indianexpress.com/article/explained/didy on traditions and values of conservation and mos-european-space-agency-hera-nsas-dart- moderation. 6149023/ ● To adopt a climate-friendly and a cleaner path than the one followed hitherto by others Q.55) Consider the following statements at corresponding level of economic regarding Intended Nationally Determined development. Contributions(INDC) of India: ● To reduce the emissions intensity of its 1. To reduce its emission intensity of GDP by GDP by 33 to 35 percent by 2030 from 50 per cent below 2005 levels by the year 2005 level. 2030. ● To achieve about 40 percent cumulative 2. To create an additional carbon sink of 2.5 to electric power installed capacity from non- 3 billion tonnes of CO2 equivalent through fossil fuel based energy resources by 2030, additional forest and tree cover by 2030. with the help of transfer of technology and 3. To achieve about 40 per cent cumulative low cost international finance, including electric power installed capacity from non- from the Green Climate Fund. fossil fuel based energy resources by 2030.

● To create an additional carbon sink of 2.5 Which of the statements given above is/are to 3 billion tonnes of CO2 equivalent correct? through additional forest and tree cover a) 1 only by 2030. b) 2 only ● To better adapt to climate change by c) Both 1 and 2 enhancing investments in development d) Neither 1 nor 2 programmes in sectors vulnerable to climate change, particularly agriculture, water Ans) c resources, Himalayan region, coastal regions, Exp) Both statements are correct. health and disaster management. The major reef formations in India are ● To mobilize domestic and new and additional restricted to the Gulf of Mannar, Palk bay, funds from developed countries to implement Gulf of Kutch, Andaman and Nicobar Islands the above mitigation and adaptation actions and the Lakshadweep islands. While the in view of the resource required and the Lakshadweep reefs are atolls, the others are all resource gap. fringing reefs. Patchy coral is present in the ● To build capacities, create domestic intertidal areas of the central west coast of the framework and international architecture for country. Coral reefs in India are being damaged quick diffusion of cutting edge climate and destroyed at an increasing rate. They face technology in India and for joint serious problems of stress from anthropogenic collaborative R&D for such future pressures and interference. technologies. Reefs can vary enormously in structure and Hence, statement 1 is incorrect but statements complexity and are roughly divided into three 2 and 3 are correct. major types. 1. Fringing reefs', reefs that grow close to the Source) shore and extend out into the sea like a https://pib.gov.in/PressReleseDetailm.aspx?PRI submerged platform. Hence statement 1 is D=1648773 incorrect. https://timesofindia.indiatimes.com/india/net- 2. Barrier reef: reefs separated from the land by zero-carbon-emitter-by-2030-railways-begins- wide expanses of water and follow the coastline. talks-with-key-stakeholders-in-solar-power- 3. Atolls: a roughly circular ring of reefs sector/articleshow/77785634.cms surrounding a lagoon, a low lying island, https://vikaspedia.in/energy/environment/climate common in the Indian and South pacific -change/india2019s-intended-nationally- oceans. Hence statement 2 is correct. determined-contribution

Q.56) In context of coral reefs, consider the following statements: 1. 1..Atoll coral reefs are a roughly circular ring of reefs surrounding a lagoon. 2. Atoll types of coral reefs are found in the Indian Ocean.

Exp) Recently India, Japan and Australia have begun discussions on launching a trilateral Supply Chain Resilience Initiative (SCRI) to reduce dependency on China, necessitated by Beijing’s aggressive political and military behaviour. The initiative was first proposed by Japan. Hence, statement 2 is incorrect.

The two-fold objective of the Japanese proposal is to attract foreign direct investment to turn the Indo-Pacific into an “economic powerhouse” and to build a mutually complementary relationship among partner countries. Hence, statement 1 is correct.

Source) Source) http://newsonair.com/News?title=India%2c- https://economictimes.indiatimes.com/news/eco Maldives-ink-contract-for-development-of-five- nomy/foreign-trade/india-japan-australia-supply- eco-tourism-zones-in-Addu-atoll-of-island- chain-in-the-works-to-counter- nation&id=396977 china/articleshow/77624852.cms?from=mdr http://www.fao.org/3/x5627e/x5627e06.htm#:~:t https://www.thehindu.com/business/Economy/w ext=The%20major%20reef%20formations%20i hat-is-the-new-idea-on-supply- n,west%20coast%20of%20the%20country. chains/article32476160.ece#:~:text=What%20do https://incois.gov.in/portal/CoralReef.jsp es%20supply%20chain%20resilience,just%20on e%20or%20a%20few. Q.57) In context of Supply Chain Resilience Initiative recently seen in the news, consider the Q.58) Consider the following: following statements: 1. Poliomyelitis 1. It aims to attract foreign direct investment to 2. Ebola turn the Indo-Pacific region into an economic 3. Elephantiasis powerhouse. Which of the above disease/diseases is/are caused 2. This initiative has been launched by the by a virus? member countries of the BRICS. a) 1 and 2 only Which of the following statements is/are not b) 1 only correct? c) 2 and 3 only a) 1 only d) 1, 2 and 3 only b) 2 only c) Both 1 and 2 Ans) a d) Neither 1 nor 2 Exp) Options 1 and 2 are correct. Poliomyelitis or Polio. It is a highly infectious Ans) b disease caused by a virus. It invades the nervous system, and can cause total paralysis in a matter

of hours. The virus is transmitted by person-to- https://www.who.int/health- person spread mainly through the faecal-oral topics/ebola/#tab=tab_1 route or, less frequently, by a common vehicle (for example, contaminated water or food) and Q.59) Consider the following statements; multiplies in the intestine. Initial symptoms are 1. The 15th Finance Commission had decreased fever, fatigue, headache, vomiting, stiffness of the forest-based devolution of funds to states the neck and pain in the limbs from 25 percent to 10 percent.

2. States can use a compensatory afforestation Ebola virus disease (EVD), formerly known as fund for the activities of wildlife Ebola haemorrhagic fever, is a severe, often fatal illness affecting humans and other primates. management, forest fire prevention. The virus is transmitted to people from wild Which of the statements given above is/are animals (such as fruit bats, porcupines and non- correct? human primates) and then spreads in the human a) 1 only population through direct contact with the blood, b) 2 only secretions, organs or other bodily fluids of c) Both 1 and 2 infected people, and with surfaces and materials d) Neither 1 nor 2 (e.g. bedding, clothing) contaminated with these fluids. The first EVD outbreaks occurred in remote Ans) b villages in Central Africa, near tropical Exp) Statement 1 is incorrect. The 15th Finance rainforests. The 2014–2016 outbreak in West Commission had increased the forest-based Africa was the largest and most complex Ebola devolution of funds from 7 per cent to 10 per cent. outbreak since the virus was first discovered in Now states will receive 10 percent of devolution of funds for their forest cover. 1976.

Elephantiasis. It is also known as lymphatic filariasis. It’s caused by parasitic worms, and can spread from person to person through mosquitoes. Elephantiasis causes swelling of the scrotum, legs, or breasts. Elephantiasis is considered a neglected tropical disease (NTD). It’s more common in tropical and subtropical areas of the world, including Africa and Southeast Asia. Hence Option 3 is incorrect.

Source) https://www.healthline.com/health/elephantiasis https://www.who.int/news-room/fact- Statement 2 is correct. The Compensatory sheets/detail/poliomyelitis Afforestation Fund Act, 2016 provides for setting https://www.who.int/denguecontrol/disease/en/ up Compensatory Afforestation Fund Management and Planning Authority (CAMPA) at both central and state levels to ensure

expeditious and transparent utilization of A nontariff barrier is a way to restrict trade using amounts realized in lieu of forest land diverted for trade barriers in a form other than a tariff. non-forest purpose. The Campa Afforestation Nontariff barriers include quotas, embargoes, Fund will be used as per provisions of the CAF sanctions, and levies. As part of their political or Act and Rules. These include catchment area economic strategy, some countries frequently use treatment, wildlife management, forest fire nontariff barriers to restrict the amount of trade prevention, soil and moisture conservation they conduct with other countries. work in the forest, however the fund cannot be used for payment of salary, travelling allowances, Non-Tariff Barriers to Trade making buildings and buying office equipment (a) Licenses. A license is granted to a business by for forest officers the government and allows the business to import a certain type of good into the country. For Source) example, there could be a restriction on imported https://www.newindianexpress.com/nation/2020 cheese, and licenses would be granted to certain /aug/22/nearly-three-fourth-of-data-related-to- companies allowing them to act as importers. compensatory-afforestation-from-states- This creates a restriction on competition and incorrect-ministry-2186946.html increases prices faced by consumers. https://indianexpress.com/article/india/project- (b) Import Quotas. An import quota is a dolphin-to-be-launched-in-a-fortnight-campa- restriction placed on the amount of a particular funds-to-only-be-used-for-plantation-drives- good that can be imported. This sort of barrier is afforestation-javadekar-6559092/ often associated with the issuance of licenses. For https://www.prsindia.org/report- example, a country may place a quota on the summaries/report-15th-finance-commission-fy- volume of imported citrus fruit that is allowed. 2020-21 (c) Voluntary Export Restraints (VER). This type of trade barrier is "voluntary" in that it is Q.60) Consider the following: created by the exporting country rather than the 1. Use of licenses to limit imported goods. importing one. A voluntary export restraint is 2. Issue of quotas for exporting of goods and usually levied at the behest of the importing services. country and could be accompanied by a 3. Restriction of Local content requirement for reciprocal VER. imports of goods and services. (d) Standards. Standards take a special place 4. Use of sanitary regulations. among non-tariff barriers. Countries usually impose standards on classification, sanitary Which of the above steps is/are included in Non regulations, labelling and testing of products to Tariff Barriers of Trade? ensure that domestic products meet domestic a) 2 and 3 only standards, but also to restrict sales of products of b) 1 and 4 only foreign manufacture unless they meet or exceed c) 1, 2 and 3 only these same standards. These standards are d) All of the above sometimes entered to protect the safety and health

of local populations and the natural environment. Ans) d (e) Local Content Requirement. Instead of Exp) All options are correct. placing a quota on the number of goods that can

be imported, the government can require that a coccolithophores, with potential significance for certain percentage of a good be made the world ocean ecosystem. domestically. The restriction can be a percentage of the good itself or a percentage of the value of Coccolithophores are single-celled algae living the good. For example, a restriction on the import in the upper layers of the world’s oceans. They of computers might say that 25% of the pieces have been playing a key role in marine used to make the computer are made ecosystems and the global carbon cycle for domestically, or can say that 15% of the value of millions of years. Coccolithophores calcify the good must come from domestically produced marine phytoplankton that produce up to 40 per components. cent of open ocean calcium carbonate and are responsible for 20 per cent of the global net Source) marine primary productivity. https://timesofindia.indiatimes.com/business/ind ia-business/difficult-quality-standard-norms-of- Coccolithophores build exoskeletons from us-eu-act-as-non-tariff-barriers-piyush- individual CaCO3 plates consisting of chalk and goyal/articleshow/77878397.cms seashells building the tiny plates on their exterior. https://www.investopedia.com/articles/economic Though carbon dioxide is produced during the s/08/tariff-trade-barrier-basics.asp formation of these plates, coccolithophores help https://en.wikipedia.org/wiki/Non- in removing it from the atmosphere and ocean by tariff_barriers_to_trade\ consuming it during photosynthesis. At equilibrium, coccolithophores absorb more carbon dioxide than they produce, which is Q.61) The term Coccolithopores recently seen in beneficial for the ocean ecosystem. the news is a/an a) fossil of oldest reptiles found in Alps, Europe. Source) b) tree dwelling species of frog. https://www.downtoearth.org.in/news/climate- c) fungal herb which can act as an aphrodisiac. change/ancient-algae-play-a-role-in-building-a- d) ancient marine algae. healthy-marine-ecosystem-study-71909

Ans) d Q.62) With reference to Seabed 2030 Project Exp) sometimes seen in the news, consider the A study of a microscopic ancient marine algae following statements: (Coccolithophores) led by the National Centre for 1. The project aims to measure the shape and Polar and Ocean Research (NCPOR) has found depth of the world ocean floor. that there is a decrease in the concentration of 2. It is a joint initiative of the United Nations oceanic calcium carbonate (CaCO3) in the Environment Programme and GreenPeace. southern Indian ocean. Which of the statements given above is/are correct? This decrease in CaCO3 is attributed to the a) 1 only increase in the concentration of another single- b) 2 only celled algae known as diatoms. This, in turn, will c) Both 1 and 2 affect the growth and skeleton structure of d) Neither 1 nor 2

Ans) a The knowledge of bathymetry — the Exp) measurement of the shape and depth of the ocean Statement 1 is correct but statement 2 is floor — is instrumental in understanding several incorrect. natural phenomena, including ocean circulation, The Seabed Project 2030 is a collaboration tides, and biological hotspots. It also provides key between Japan’s non-profit Nippon Foundation inputs for navigation, forecasting tsunamis, and the General Bathymetric Chart of the Oceans exploration for oil and gas projects, building (GEBCO). The Nippon Foundation-GEBCO offshore wind turbines, fishing resources, and for Seabed 2030 Project, which is coordinating laying cables and pipelines. efforts to complete the mapping of the entire ocean floor by 2030. Source) https://indianexpress.com/article/explained/map A bathymetric chart is a type of isarithmic map ping-the-ocean-floor-6473482/ that depicts the submerged topography and https://www.jatinverma.org/why-scientists- physiographic features of ocean and sea bottoms. want-to-map-the-entire-ocean-floor Their primary purpose is to provide detailed depth contours of ocean topography as well as Q.63) Consider the following: provide the size, shape and distribution of River water dispute Between states underwater features. 1. Vamsadhara Andhra Pradesh and Odisha GEBCO is the only intergovernmental 2. Krishna Karnataka and Telangana organisation with a mandate to map the entire 3. Mahdayi Maharashtra and Goa ocean floor. Which of the pairs given above is/are correctly matched? The Seabed Project was launched at the United a) 1 and 2 only Nations Ocean Conference in 2017, and b) 2 and 3 only coordinates and oversees the sourcing and c) 1 only compilation of bathymetric data from different d) 1, 2 and 3 parts of the world’s ocean. For this, the project aims to rope in governments, Ans) d private companies, and international Exp) All pairs are correctly matched. organisations to acquire data. ● Krishna Water Dispute - Karnataka, Significance of the project: Telangana, Andhra Pradesh and Maharashtra ● In the past, satellites and planes carrying ● Mahadayi Water Dispute - Goa, Karnataka altimeter instruments have been able to and Maharashtra provide large swathes of data about the ocean floor. ● Vansadhara Water Dispute - Andhra Pradesh ● The Seabed 2030 Project, however, aims to & Odisha obtain higher quality information, using Source) equipment such as deepwater hull-mounted sonar systems, and more advanced options https://www.thehindu.com/news/national/andhra such as Underwater Vehicles (AUVs). -pradesh/jagan-will-take-up-vamsadhara-

waterrow-with-naveen-soon- https://www.financialexpress.com/economy/east minister/article31769351.ece -asia-summit-india-china-us-to-bolster-supply- chain-to-deal-with-challenges-posed-by- https://pib.gov.in/PressReleasePage.aspx?PRID covid/2069325/ =1556784 https://www.dfat.gov.au/international- relations/regional-architecture/eas/Pages/east- Q.64) Consider the following countries: asia-summit-eas 1. India Q.65) Consider the following statements 2. USA regarding Etikoppaka Toys, recently seen in the 3. China news: 4. Russia 1. These toys are made up of wood and lacquer 5. Germany color. 6. Srilanka 2. These toys were round from all sides with no Which of the above is/are not a member of the corners. East Asia Summit ? 3. Etikoppaka Toys owes its origin to the state a) 2, 3 and 4 only of Andhra Pradesh. b) 2, 3, 5 and 6 only Which of the statements given above is/are c) 5 and 6 only correct? d) 5 only a) 1 and 2 only b) 2 and 3 only Ans) c c) 1 and 3 only Exp) The East Asia Summit (EAS) is the Indo- d) 1, 2 and 3 Pacific's premier forum for strategic dialogue. It is the only leader-led forum at which all key Ans) d partners meet to discuss the full range of political, Exp) All statements are correct. security and economic challenges facing the Etikoppaka is a small village on the banks of Indo-Pacific, and has an important role to play in Varaha River in Visakhapatnam district of advancing closer regional cooperation. Australia Andhra Pradesh.In 2017, the Etikoppaka toys participated, as a founding member, in the acquired a Geographical Indications(GI) tag. inaugural EAS held in Kuala Lumpur on 14 The speciality of these toys - December 2005. ● These toys are made of wood and lacquer The EAS has 18 members - the ten ASEAN color. These toys coloured with natural dyes countries (Brunei, Cambodia, Indonesia, Laos, derived from seeds,lacquer,roots and Malaysia, Myanmar, the Philippines, Singapore, leaves.The way of toy making is also known Thailand, Vietnam) along with Australia, China, as turned wood lacquer craft. India, Japan, New Zealand, the Republic of ● One would not find any angles or corners Korea, the United States and Russia. in these toys anywhere. These toys were (Germany and Srilanka are not the member of round from all sides hence there was no EAS, Hence option 5 and 6 are incorrect) scope for injury to children.

Source)

Source) atoms become a single entity with quantum https://www.deccanchronicle.com/nation/politic properties, wherein each particle also functions as s/300820/pm-hails-etikoppaka-toy-maker-cv- a wave of matter. raju-in-mann-ki-baat-bats-for-local.html https://www.thehindu.com/entertainment/art/etik BECs straddle the line between the macroscopic oppaka-toys-of-andhra-pradesh-in-the- world governed by forces such as gravity and the spotlight/article32496524.ece microscopic plane, ruled by quantum mechanics.

BECs are extremely fragile. The slightest Q.66) Recently, the the fifth state of matter Bose interaction with the external world is enough to Einstein Condensates was seen in the news, it will warm them past their condensation threshold. help in This makes them nearly impossible for scientists 1. Searches for dark energy and Gravitational to study on Earth, where gravity interferes with waves. the magnetic fields required to hold them in place 2. Spacecraft navigation. for observation. 3. Prospecting for subsurface minerals on the moon. Applications of BECs: • Tests of general relativity Which of the statements given above is/are • Searches for dark energy and correct? gravitational waves. a) 1 and 2 only • Spacecraft navigation b) 2 and 3 only • Prospecting for subsurface minerals on the c) 1 and 3 only moon and other planetary bodies. d) All of the above • One of the most interesting applications is

Matter lasers, as Bose-Einstein condensate Ans) d exhibits qualities of coherent matter, much Exp) All statements are correct. like the photons in current lasers. Scientists have observed the fifth state of matter in space for the first time, offering unprecedented insight that could help solve some of the quantum Source) https://www.ndtv.com/science/bose- universe's most intractable conundrums. Solids, einstein-quantum-fifth-state-of-matter-seen-for- liquids, gases and plasma are the other four states 1st-time-in-space-2244587 of matter. Q.67) With reference to Malabar uprisings BEC is a supercooled gas that no longer behaves recently seen in the news, consider the following as individual atoms and particles, but rather an statements: entity in a single quantum state. Bose-Einstein 1. It was an armed revolt staged by the Mappila condensates (BECs) -- the existence of which was Muslims of Kerala against their Hindu predicted by Albert Einstein and Indian landlords. mathematician Satyendra Nath Bose almost a 2. The revolt happened within the broad century ago -- are formed when atoms of certain spectrum of Direct Action day organised by elements are cooled to near absolute zero (0 the Muslim League. Kelvin, minus 273.15 Celsius). At this point, the

Which of the statements given above is/are overtones that culminated into communal correct? violence. a) 1 only b) 2 only Direct Action Day (16 August 1946), also c) Both 1 and 2 known as the 1946 Calcutta Killings, was a day d) Neither 1 nor 2 of widespread communal rioting between Muslims and Hindus in the city of Calcutta (now Ans) a known as Kolkata) in the Bengal province of Exp) Statement 1 is correct. The Malabar British India. The day also marked the start of rebellion, also known popularly as the Moplah what is known as The Week of the Long Knives. rebellion, was an armed revolt staged by the Mappila Muslims of Kerala against the British Source) https://theprint.in/india/why-1921- authorities and their Hindu landlords in 1921. malabar-moplah-rebellion-wasnt-a-peasant- The six-month-long rebellion is often perceived uprising-but-an-anti-hindu-genocide/485065/ to be one of the first cases of nationalist uprisings https://www.thehindu.com/news/national/kerala/ in Southern India. The uprising occurred within an-artefacts-link-to-a- the broader spectrum of the Khilafat/Non- rebellion/article31948690.ece cooperation movement led by Mahatma https://eduindexnews.com/2020/07/20/malabar- Gandhi. Hence statement 2 is incorrect. rebellion-looking-100-years-back/ https://www.jatinverma.org/variyamkunnath- About Rebellion.The rebellion largely took the kunjahammed-haji-the-khilafat-leader shape of guerrilla-type attacks on janmis, police and troops, Symbols of the colonial state – Q.68) Consider the following statements telegraph lines, train stations, courts, post offices regarding Regional Comprehensive Economic etc. and homes of landlords were attacked. When Partnership (RCEP): the rebellion spread across the Malabar district, 1. It is a proposed Agreement between ASEAN British officers and the local police escaped, nations and their FTA partners. leaving vast tracts of territory firmly under the 2. India is a member of RCEP. control of the local rebels. Which of the statements given above is/are Mappilas under the leadership of correct? Variyamkunnath Kunjahammed Haji took up a) 1 only arms in August 1921 due to the arrest of Khilafat b) 2 only leader Ali Musaliyar and a widespread rumour c) Both 1 and 2 that a prominent mosque in Thirurangadi has d) Neither 1 nor 2 been raided. For nearly six months, he ran a parallel Khilafat regime headquartered in Ans) a Nilambur, with even its own separate passport, Exp) The Regional Comprehensive Economic currency and system of taxation. Tenants were Partnership (RCEP) is a proposed agreement granted the power over the lands they cultivated between the member states of the Association of along with tax incentives and although the Southeast Asian Nations (Asean) and its free movement started off largely as a protest against trade agreement (FTA) partners. The pact aims to British authorities, it acquired communal

cover trade in goods and services, intellectual Exp) China has begun piloting its new digital property, etc. (Statement 1 is correct) currency in four major cities, becoming the first RCEP aims to create an integrated market with 16 nation to do so. The nation has reportedly begun countries, making it easier for products and trials in four cities, including Shenzhen, Suzhou, services of each of these countries to be available Chengdu as well as Xiong’an, and areas that will across this region. host some of the events for the 2022 Beijing Winter Olympics. It has been formally adopted The negotiations are focused on the following: into the cities’ monetary systems. Trade in goods and services, investment, intellectual property, dispute settlement, e- The digital currency–known as the e-RMB– commerce, small and medium enterprises, and would be used for paying the April-month economic cooperation. salaries to some government employees and public servants. The digital currency will be used In a significant development the current members to subsidise transport in Suzhou, but in Xiong’an of Regional Comprehensive Economic the trial primarily focused on food and retail. Partnership (RCEP) in the backdrop of Covid that (Option b is correct) has wreaked havoc on economies has stated that it would welcome India’s return to the Source) https://www.theweek.in/news/biz- negotiating table for entering the trade bloc. tech/2020/05/01/amid-covid-backlash-china- (Statement 2 is incorrect) tests-state-run-digital-currency-to-counter- dollar-monopoly.html Source) https://www.business- https://www.thehindubusinessline.com/opinion/c standard.com/about/what-is-rcep olumns/the-cheat-sheet/what-chinas-digital- https://economictimes.indiatimes.com/news/eco currency-means-for-the- nomy/policy/rcep-urges-india-to-return-to- world/article32337272.ece negotiating-table-as-covid-19-wrecks- economies/articleshow/75477407.cms?from=md Q.70) Consider the following statement r#:~:text=Synopsis&text=New%20Delhi%3A% regarding Saffron: 20In%20a%20significant,for%20entering%20th a) Kashmiri Saffron is the only saffron in the e%20trade%20bloc. world grown at an altitude of 1,600 m to 1,800 m above mean sea level. Q.69) Which of the following best describes “e- b) India is the largest producer of saffron in the RMB” recently seen in the news? world. a) It is a test to detect COVID-19 virus. Which of the statements given above is/are b) It is the proposed digital currency of China. correct? c) It is a proposed standard compliance mark for a) 1 only all electric vehicles in India. b) 2 only d) It is an improved form of e-cigarette with less c) Both 1 and 2 percentage of nicotine. d) Neither 1 nor 2

Ans) a Ans) b

Exp) Kashmir saffron, which is cultivated and Q.71) Consider the following statements harvested in the Karewa (highlands) of Jammu regarding ‘Year of awareness on science and and Kashmir, has been given the Geographical health’ (YASH) 2020-2021: Indication (GI) tag by the Geographical 1. It aims at sharing the best practices and habits Indications Registry. for good health and hygiene. 2. It aims to promote scientific temperament Kashmir saffron is a very precious and costly among the masses through community product. Iran is the largest producer of saffron participation at the grassroots. and India is a close competitor. With the GI tag, 3. It has been launched by the Ministry of Kashmir saffron would gain more prominence in Health and Family Welfare, Government of the export market. (Statement 2 is incorrect) India. Which of the statements given above is/are It is the only saffron in the world grown at an correct? altitude of 1,600 m to 1,800 m AMSL (above a) 1 and 2 only mean sea level), which adds to its uniqueness and b) 2 and 3 only differentiates it from other saffron varieties c) 1 and 3 only available the world over. (Statement 1 is d) 1, 2 and 3 correct) Ans) a The saffron available in Kashmir is of three types Exp) In the wake of the coronavirus (COVID-19) — ‘Lachha Saffron’, with stigmas just separated outbreak, the Department of Science and from the flowers and dried without further Technology (DST) has decided to launch a year- processing; ‘Mongra Saffron’, in which stigmas long health and science awareness programme are detached from the flower, dried in the sun and based on the pandemic. (Statement 3 is processed traditionally; and ‘Guchhi Saffron’, incorrect) which is the same as Lachha, except that the latter’s dried stigmas are packed loosely in air- The programme will aim at sharing the best tight containers while the former has stigmas practices and habits for good health and hygiene joined together in a bundle tied with a cloth and promoting scientific temperament among the thread. masses through community participation at the grassroots. (Statements 1 and 2 are correct) Saffron cultivation is believed to have been introduced in Kashmir by Central Asian The National Council for Science and immigrants around the 1st Century BCE. In Technology has decided to observe 2020-2021 as ancient Sanskrit literature, saffron is referred to the ‘Year of awareness on science and health’ as ‘bahukam’. (YASH). Since the outbreak, there has been significant Source) emphasis on personal hygiene, need for washing https://www.thehindu.com/news/national/other- hands with soap, maintaining physical distancing, states/kashmir-saffron-gets-gi- curbs on open spitting among many other tag/article31484569.ece preventive measures. However, with many of these being “uncommon” practices for an average

Indian, activities under YASH target minimising for the subsequent six months, as per a health risks at all levels with public notification issued by the Finance Ministry communication and outreach. The programme will be a platform to promote Safeguard duties can be imposed on items, public understanding of common minimum over and above existing customs duties, if it can science for community care and health safety be conclusively proved that a steep increase in measures, which include some of these common imports over a period of time resulted in injury habits. and disruption for local businesses. (Statements The programme also aims at inculcating scientific 1 and 2 are correct) temper for adopting sustainable lifestyle, nurturing scientific culture among people. Source) Source) https://www.thehindubusinessline.com/economy https://indianexpress.com/article/india/maharash /policy/finance-ministry-extends-safeguard- tra-govt-plans-community-outreach-for-better- duty-on-import-of-solar-cells- health-hygienic-practices-6393899/ modules/article32224865.ece#:~:text=Safeguard %20duties%20can%20be%20imposed,and%20d Q.72) Consider the following statements isruption%20for%20local%20businesses. regarding “Safeguard Duty”: 1. It can be imposed on items, over and above Q.73) Which of the following is the aim of existing customs duties. “BOBBLE Project” recently seen in the news? 2. It can be imposed if it is conclusively proved a) It aims to provide internet connectivity in that a steep increase in imports over a period remote areas using high-altitude balloons in of time resulted in injury and disruption for the stratosphere. local businesses. b) It aims to determine, quantify and model Which of the statements given above is/are ocean-atmosphere interactions that drive correct? variability in the south Asian monsoon. a) 1 only c) It aims to use rocket technology for b) 2 only transportation of passengers from one place c) Both 1 and 2 to another in a very short period of time. d) Neither 1 nor 2 d) It aims to detect black holes and gravitational waves nearest to our solar system. Ans) c Exp) The Finance Ministry has extended the safeguard duty on imported solar cells and solar Ans) b modules by another year as recommended by the Exp) BoBBLE or Bay of Bengal Boundary Layer Director-General of Trade Remedies (DGTR) Experiment, is to determine, quantify and model earlier this month. ocean-atmosphere interactions that drive variability in the south Asian monsoon. (Option A safeguard duty rate of 14.90 per cent will be b is correct.) imposed on solar cells and modules starting for the first six months starting July 30 (minus anti- BoBBLE is led by UEA scientists in dumping duty payable, if any) and 14.50 per cent collaboration with the University of Reading and the National Oceanography Centre in

Southampton. Collaborators in India include the Dealing with the COVID-19 crisis has been a Centre for Atmospheric and Oceanic Sciences, challenge for most countries across the globe the Indian National Centre for Climate which has witnessed some sort of lockdown in Information Services, the National Centre for most of the places. While the general narrative Medium Range Weather Forecasting and the around this crisis has been that of anxiety and National Institute of Ocean Technology. concern, the crisis has also thrown up some positive developments. One of these is the visible Source) improvement in the natural environment. Rivers https://www.livemint.com/Politics/st72RvH8yR have become cleaner. The air has become fresher. 6vDk3i6dPldN/UK-India-researchers-to-use- There has been a significant drop in GHG underwater-robots-and-specialis.html emissions. Animals and birds are returning to and https://www.thehindu.com/news/cities/Kochi/a- enjoying their habitats. Purely from a river blueprint-for-future-weathersystem- management point of view, in India there has observational-experiments/article31511817.ece been a noticeable improvement in the water quality of the Ganga and Yamuna in the last few Q.74) Consider the following statements weeks. During the last year or so, the Gangetic regarding IDEAthon on “The future of River Dolphin, an indicator species, has been showing Management’ recently seen in the news: improvements with sightings at several stretches 1. It aimed to explore how the COVID-19 crisis of the river. The sighting of this is more frequent could shape River Management strategies for during lockdown in Ganga and its tributaries. the future. Venice’s (in) famously polluted canals have 2. It was organised by the National Mission for become clearer as tourists stay away. For the first Clean Ganga (NMCG) under the Ministry of time in recent history, dolphins are back in the Jal Shakti and National Institute of Urban waterways of Italy as navigation has stopped. Affairs (NIUA). Which of the statements given above is/are Source) correct? https://pib.gov.in/PressReleasePage.aspx?PRID a) 1 only =1620441 b) 2 only c) Both 1 and 2 Q.75) Consider the following statements d) Neither 1 nor 2 regarding “BharatMarket”, sometimes seen in the news: Ans) c 1. It is a proposed national e-commerce Exp) The National Mission for Clean Ganga marketplace for all retail traders. (NMCG) under the Ministry of Jal Shakti and 2. It will integrate capabilities of various National Institute of Urban Affairs (NIUA) technology companies to provide end-to-end organized an IDEAthon on “The future of River services in the logistics and supply chains Management’ to explore how the COVID-19 from manufacturers to end consumers. crisis can shape River Management strategies 3. It will be launched by the Ministry of for the future. (Statements 1 and 2 are correct) Commerce and Industry, Government of India.

Which of the statements given above is/are Q.76) Which of the following is the objective of correct? the “PRAAPTI Portal”? a) 1 and 2 only a) To bring in transparency in distribution of b) 2 and 3 only food grains under National Food Security Act c) 1 and 3 only b) To bring in transparency in power purchase d) 1,2 and 3 transactions between the generators and discoms. Ans) a c) To bring in transparency in distribution of old Exp) Traders’ body Confederation of All India age pension to senior citizens Traders (CAIT) (Statement 3 is incorrect) said d) To bring in transparency in distribution of that it will soon launch a national e-commerce money under PM Kishan Samman Nidhi marketplace ‘BharatMarket’ for all retail traders Yojana in collaboration with several technology partners. The marketplace will integrate capabilities of Ans) b various technology companies to provide end-to- Exp) PRAAPTI stands for Payment Ratification end services in the logistics and supply chains And Analysis in Power procurement for bringing from manufacturers to end consumers, including Transparency in Invoicing of generators. The deliveries at home.The e-commerce portal will PRAAPTI portal was launched in May 2018 to include a nationwide participation by retailers. bring in transparency in power purchase (Statements 1 and 2 are correct) transactions between the generators and discoms. (Option b is correct) This endeavour aims to bring 95 per cent of retail traders on board the platform, who will be the Power producers' total outstanding dues owed by shareholders and the portal will be run distribution firms rose over 47 per cent year-on- exclusively by the traders. They have already year to Rs 1.33 lakh crore in June 2020, reflecting started this programme as a pilot, initially with a stress in the sector. limited number of essential commodities, in six cities — Prayagraj, Gorakhpur, Varanasi, Distribution companies (discoms) owed a total of Lucknow, Kanpur and Bengaluru, with Rs 90,655 crore to power generation firms in June tremendous response from retailers, distributors 2019, according to portal PRAAPTI (Payment and even consumers. “This has now grown to Ratification And Analysis in Power procurement over 90 cities in a matter of two weeks. The for bringing Transparency in Invoicing of learnings from the pilot will allow them to scale generators). to many more locations and soon to categories beyond groceries. Source) https://www.livemint.com/industry/energy/rs- Source) 68k-cr-of-loans-released-under-discoms- https://www.financialexpress.com/industry/bhar liquidity-package-11596969574484.html atmarket-cait-to-launch-e-commerce-platform- https://www.newindianexpress.com/business/20 for-retail-traders-soon/1945808/ 20/aug/02/discoms-outstanding-dues-to-power- producers-rise-47-per-cent-to-rs-133-lakh-crore- in-june-2178075.html

b) It is a trawler invented by Russia to cut ice Q.77) Consider the following statements through the Arctic. regarding “UV Blaster” recently seen in the c) It is a series of satellites to be launched by news: Russia for monitoring Arctic Climate. 1. It is an ultra violet (UV) disinfection tower d) It is a ship developed by China which can for rapid and chemical-free disinfection of glide on Arctic ice. high infection-prone areas. 2. It was developed by Defence Research and Ans) c Development Organisation (DRDO). Exp) Russia will launch the first Arktika-M Which of the statements given above is/are satellite for monitoring Arctic climate this correct? year at the end of the year. As of now the first a) 1 only satellite is developed and the launch is planned b) 2 only for 2020. The satellite will be launched by Soyuz c) Both 1 and 2 -2,1b carrier rocket with frigate booster.The d) Neither 1 nor 2 remote sensing Artika-M will monitor the meteorological conditions in the polar region Ans) c Exp) The Defence Research and Development Source) Organisation (DRDO) has developed an ultra http://newsonair.com/News?title=Russia-to- violet (UV) disinfection tower for rapid and launch-first-satellite-to-monitor-Arctic-climate- chemical-free disinfection of high infection- this-year&id=387614 prone areas. (Both statements are correct) Q.79) Consider the following statements The equipment named UV Blaster is “useful for regarding GARUD Portal: high tech surfaces like electronic equipment, 1. It aims to provide fast-track approval to computers and other gadgets in laboratories and COVID-19-related drone operations. offices that are not suitable for disinfection with 2. It was launched by the Ministry of Science chemical methods. The product is also effective and Technology, Government of India. for areas with large flow of people such as Which of the statements given above is/are airports, shopping malls, metros, hotels, factories correct? and offices. a) 1 only b) 2 only Source) c) Both 1 and 2 https://www.thehindu.com/news/national/drdo- d) Neither 1 nor 2 develops-uv-disinfection-tower-for-sanitising- coronavirus-prone-areas/article31503039.ece Ans) a Exp) The Civil Aviation Ministry and the Q.78) Which of the following best describes Directorate General of Civil Aviation (DGCA) Arktika-M recently seen in the news? launched GARUD portal for fast-track approval a) It is a meteorological station established by to COVID-19-related drone operations. India in the Arctic region to study its climate. (Statement 1 is correct but statement 2 is incorrect.)

The objective of the GARUD (Government The move comes after India faced quality issues Authorisation for Relief Using Drones) portal is with Chinese-made antibody testing kits, which to assist state-owned entities in seeking are used for disease surveillance and to find out exemption from central government for how many people have developed immunity to a coronavirus related drone operations. disease.

This will also help in addressing the challenges Project CARD’s first goal is to roll out at least 10 posed by COVID-19 and will remain in force million rapid antibody tests for Covid-19 by July. until further orders. Any violation of provisions Additionally, capacity will be expanded to make will make the conditional exemption null and reverse transcription polymerase chain reaction void and will lead to penal action. Getting (RT-PCR) and other paper-based tests for Covid- necessary approvals from the competent 19 in the country. authority & launching the portal in less than two weeks is a testimony of the hard work & Source) dedication of officials at MoCA, DGCA, AAI https://economictimes.indiatimes.com/industry/h and NIC. ealthcare/biotech/project-card-to-push-local- production-of-testing- Source) kits/articleshow/75585767.cms?from=mdr https://www.businesstoday.in/current/economy- politics/dgca-launches-garud-portal-to-fast- Q.81) Which of the following best describes the track-exemptions-of-coronavirus-related-drone- term “Solidarity Trial” sometimes seen in the operations/story/402956.html news? a) It is an international arrangement to express Q.80) Which of the following is the objective of solidarity towards those affected by natural “Project CARD” recently seen in the news? disaster globally. a) To scale up protection and preservation of b) It is an international clinical trial meant to soil in India help find an effective treatment for Covid-19. b) To scale up effort for early rollout of one c) It is an arrangement to express remorse for a nation one ration card gruesome act committed by a head of the c) To disseminate awareness among people to nation. limit number of invitee in marriage during d) It is an agreement under which a country can corona put on trial an accused for a crime committed d) To scale up India’s capacity to make in the territorial jurisdiction of another coronavirus testing kits. country.

Ans) d Ans) b Exp) Niti Aayog and the Department of Exp) The Solidarity trial, an international clinical Biotechnology have launched the Consortium for trial meant to help find an effective treatment for Affordable & Rapid Diagnostics (CARD) to Covid-19, compares options against standard of scale up India’s capacity to make coronavirus care and assesses their relative effectiveness testing kits. (Option d is correct.) against the disease. (Option b is correct)

As of 1 July 2020, nearly 5,500 patients have While 23 tigers were found in 24 Parganas been recruited in 21 countries among the 39 (South) Division, 73 big cats tigers were recorded countries that have approvals to begin recruiting. inside the four divisions of the Sunderban Tiger Overall, over 100 countries in all six WHO Reserve. regions have joined or expressed an interest in joining. By enrolling patients in multiple Estimation of the number of tigers in the countries, the trial aims to rapidly discover Sunderbans, a world heritage as well as a whether any of the drugs slow disease Ramsar site, has always been a challenge progression or improve survival. because of the difficult terrain that comprises dense mangrove forests, with creeks and rivulets, Source) https://theprint.in/health/this-is-how- and floods twice a day during the high tides. india-set-up-whos-drug-solidarity-trial-in-less- (Statement 2 is correct.) than-a-month-despite-lockdown/473741/ Source) https://www.thehindu.com/life-and- Q.82) Consider the following statements style/tiger-population-in-sunderbans-rises-to- regarding Sunderbans Delta: 96/article31529772.ece 1. It is the only mangrove forest in the world inhabited by tigers. Q.83) Consider the following statements 2. It is both a world heritage site and a Ramsar regarding “Sample Registration System” recently site. seen in the news: Which of the statements given above is/are 1. It is a demographic survey for providing correct? reliable annual estimates of infant mortality a) 1 only rate, birth rate, death rate and other fertility b) 2 only and mortality indicators. c) Both 1 and 2 2. It is conducted by Ministry of Health and d) Neither 1 nor 2 Family Welfare, Government of India Which of the statements given above is/are Ans) c correct? Exp) The latest estimation of tiger numbers in the a) 1 only Indian Sundarbans indicate an increase in the b) 2 only population of big cats. According to the West c) Both 1 and 2 Bengal Forest Department, the tiger count for the d) Neither 1 nor 2 year 2019-20 rose to 96, from 88 in 2018-19. Ans) a The Sunderbans delta, spread over India and Exp) The Sample Registration System is a Bangladesh, is the only mangrove forest in the demographic survey for providing reliable annual world inhabited by tigers. (Statement 1 is estimates of infant mortality rate, birth rate, death correct.) rate and other fertility and mortality indicators at the national and sub-national levels.(Statement 1 The Sundarban mangrove forest is spread over is correct) 2,585 sq. km and includes the Sundarban Tiger Reserve and the 24 Parganas (South) Division.

Initiated on a pilot basis by the Registrar For MSMEs to take support of the government General of India in a few states in 1964-65, it schemes, Champion Control Rooms have been became fully operational during 1969-70. set up at MSME Development Institutes across (Statement 2 is incorrect) the country that will handhold small businesses in accessing help. The control rooms network is The field investigation consists of continuous created in a “Hub & Spoke Model. The Hub is enumeration of births and deaths in selected situated in New Delhi in the Secretary MSME’s sample units by resident part-time enumerators, office. The spokes will be in the states in various generally anganwadi workers and teachers; and offices and institutions of the Ministry. As of an independent retrospective survey every six now, 66 state level control rooms are created as months by SRS supervisors. The data obtained by part of the system. these two independent functionaries are matched. Source) https://www.financialexpress.com/industry/sme/ Source) msme-eodb-govt-launches-champions-portal-to- https://indianexpress.com/article/india/national- register-and-resolve-msme-grievances/1956458/ infant-mortality-rate-at-32-madhya-pradesh- worst-performer-6402497/ Q.85) In India, other than Chambal river a tourist would be able to come across Gharial in which of Q.84) Which of the following is the objective of the following rivers? “CHAMPIONS Portal” recently seen in the 1. Girwa news? 2. Ramganga a) It aims to make athletes ready for the next 3. Sone two olympics. Select the correct answer from the codes given b) It aims to utilise the experience of privately below. practicing doctors in tackling COVID-19 a) 1 only pandemic. b) 2 and 3 only c) It aims to encourage students to pursue c) 1 and 3 only careers as researchers in Pure Science. d) 1, 2 and 3 d) It aims to make a smaller MSME unit into a big company by solving their grievances, and Ans) d handholding. Exp) Forty gharials (Gavialis gangeticus) were released in the river by the Bahraich Ans) d forest division of Uttar Pradesh amid the Exp) The Champions (Creation and Harmonious nationwide lockdown due to the novel Application of Modern Processes for Increasing coronavirus disease (COVID-19). the Output and National Strength) portal is “basically for making the smaller units big by The animals were brought from the Kukrail solving their grievances, encouraging, Gharial Rehabilitation Centre in Lucknow after supporting, helping and handholding. It is a being tagged. real one-stop-shop solution of MSME Ministry. (Option d is correct.) “The Ghaghara acts as an important aquatic corridor for gharials in Uttar Pradesh. The river is

a major left bank tributary of the Ganges. This Ans) a year, eight males and 32 females were released Exp) Gharials are critically endangered in the Option 1 is correct. Originating in Tamil Nadu, International Union for Conservation of silambam, is one of the oldest self-defence Nature Red List of Species. The species is also techniques out there. One can even find listed under Schedule I of the Wild Life references of this ancient form of martial arts in (Protection) Act, 1972. TSA is involved in the Sangam Literature. Silambam mainly various conservation works across the country. involves stick fighting and was successfully used against the British army in the 18th century. A major chunk of gharials in India is found in the Eventually, the fighting technique evolved into Chambal river, which has about 1,000 adults. performance art, almost like jallikattu, and was However, there are satellite populations of less showcased on occasions like Pongal. than 100 adults in the Girwa river (Katarniaghat Wildlife Sanctuary in Uttar Option 2 is correct. Pradesh), the Ramganga river in Jim Corbett Kalaripayattu is the oldest dance form in India National Park and the Sone river. (All and originated in Kerala, it gets its name from the statements are correct) words Kalari (battleground)and Payattu (fight). “Tracking during post monsoon helps us to Believed to be the oldest-known martial arts in understand the dispersal and survival of the the country, it is also arguably the most animals. Also, it will aid our understanding recognised. While the origins of this combat regarding the percentage of wild population technique are unknown, the gap is filled by versus gharials released after captive breeding religious myths. The story goes that it was taught by Lord Shiva to Parasurama (a disciple of Lord Source) Vishnu), who then passed on the knowledge to his https://www.downtoearth.org.in/news/wildlife- 21 students. Kalaripayattu soon spread across the biodiversity/40-gharials-released-in-ghaghara- state of Kerala as Parasurama opened kalaris river-amid-lockdown- throughout the state. Another school of thought 71119#:~:text=Forty%20gharials%20(Gavialis attributes the origin to Sage Agastya and %20gangeticus)%20were,disease%20(COVID% Buddhist monk Bodhidharma. 2D19). Option 3 is incorrect. Himachal Pradesh’s Q.86) Consider the following pairs: thoda, relies on one’s archery prowess. Martial art State Considered more of a dance form than a martial 1. Silambam Tamil Nadu art technique, it was performed by the Rajput 2. Kalaripayattu Kerala warriors in the districts of Sirmour, Shimla and 3. Thoda Manipur Solan. The origins of the sport date back to the Which of the above pairs is/are correctly great epic of Mahabharata. Played between two matched? teams called ‘paasha’ and ‘saatha’, who are a) 1 and 2 only believed to be the descendants of Pandavas and b) 2 and 3 only Kauravas, thoda is organised during Baisakhi. c) 1 and 3 only d) 1, 2 and 3

Source) of Swadeshi goods and mass agitation in the https://www.outlookindia.com/outlooktraveller/e aftermath of the controversial Partition of xplore/story/70664/5-lesser-known-forms-of- Bengal in 1905 by Lord Curzon. martial-arts-in-india https://www.outlookindia.com/outlooktraveller/e Lala Lajpat Rai also wrote extensively in English xplore/story/70193/discover-the-ancient-martial- and Urdu. His important works include: ‘The arts-of-south-india Arya Samaj’, ‘Young India’, ‘England’s Debt to https://www.thehindu.com/life-and- India’, ‘Evolution of Japan’, ‘India’s Will to style/fitness/the-ancient-martial-art-of- Freedom’, ‘Message of the Bhagavad Gita’, kalaripayattu-in-delhi/article28136442.ece ‘Political Future of India’, ‘Problem of National Education in India’, ‘The Depressed Glasses’, Q.87) Consider the following statements and the travelogue ‘United States of America’. regarding Lala Lajpat Rai: 1. He advocated the use of Swadeshi goods and Statement 2 is incorrect. Lala lajpat Rai was mass agitation in the aftermath of the elected President of the Indian National Congress Partition of Bengal in 1905. during its Special Session in Kolkata in 1920, 2. He never became the president of Indian which saw the launch of Mahatma Gandhi’s Non- National Congress. cooperation Movement. He was subsequently 3. ‘Evolution of Japan’ and ‘India’s Will to imprisoned from 1921 and 1923. Rai was among Freedom’ are some of his famous works. the leaders of the movement opposing the Simon Which of the statements given above is/are Commission and was severely lathi-charged correct? during a protest in Lahore on October 30, 1928. a) 1 and 2 only b) 1 and 3 only Source) c) 2 and 3 only https://timesofindia.indiatimes.com/city/amrit d) All of the above sar/punjab-flag-row-at-lala-lajpat-rais- village/articleshow/77586729.cms Ans) b https://indianexpress.com/article/explained/britis Exp) Statements 1 and 3 are correct. Lala h-rule-remembering-lala-lajpat-rai-on-his-death- Lajpat Rai, the firebrand Indian nationalist leader anniversary-6124363/ affectionately called ‘Punjab Kesari’. Rai is remembered for his role during the Swadeshi movement and for his advocacy of education. The Q.88) Recently the term “INDRA 2020”was seen patriot died in Lahore in 1928 after he was in the news, it is attacked by police during a protest rally against a) an android malware that replaces portions of the Simon Commission. apps with its own code. b) the World’s first privately funded lunar During the Lahore Session of the Congress in space mission Mission. 1893, Rai met Bal Gangadhar Tilak, another c) a Voice over Internet Protocol service of nationalist, and the two became lifelong BSNL. associates. Rai, Tilak, and Bipin Chandra Pal d) a bilateral naval exercise between India and (called Lal-Bal-Pal) fervently advocated the use Russia.

Ans) d The concept of Zero FIR was introduced by the Exp) Recently India and Russia are scheduled to recommendation of the Justice Verma Committee hold the bilateral naval exercise, Indra 2020, in in the Criminal Law Amendment Act, 2013 after the Andaman Sea, close to the strategic Strait of the horrifying Nirbhaya rape case in Delhi. Malacca. It states that no police officer can refuse to lodge an FIR if the crime has occurred outside its The Indra series of exercises began in 2003 and jurisdiction. It is called 'zero' because a regular was conducted as a bilateral naval exercise FIR comes with a serial number, whereas this one alternately between the two countries. However, doesn't. the first joint Tri-Services Exercise was It remains a Zero FIR till a preface investigation conducted in 2017. is done, and then transferred to the police station with competent jurisdiction. Source) The Zero FIR aims to ensure evidence in cases https://economictimes.indiatimes.com/news/defe like murder, sexual assault or accident is not nce/indian-navy-to-hold-mega-exercise-with- tampered or contaminated. russia-in-bay-of-bengal-on-september-4- 5/articleshow/77875520.cms Source) https://www.financialexpress.com/defence/exerc https://www.freepressjournal.in/india/what-is- ise-indra-2020-in-the-andaman-sea-starts-friday- zero-fir-rhea-chakrabortys-legal-move-has-a- forces-of-india-russia-to-test- nirbhaya-rape-case-connection interoperability/2073622/ https://telanganatoday.com/patna-fir-best-be- regarded-as-zero-fir-and-transferred-to-mumbai Q.89) Which of the following best describes the https://timesofindia.indiatimes.com/blogs/legally term “Zero FIR” sometimes seen in the news? -speaking/zero-fir-a-potent-measure-for-prompt- a) A complaint filed in a police station and and-easy-access-to-justice/ which is yet to be registered as FIR. b) An FIR filed in a Police Station against a Q.90) Consider the following: crime committed outside its territorial 1. Child mortality jurisdiction. 2. Assets of the Household c) An FIR in which the complainant is yet to 3. Cooking Fuel produce any evidence. 4. Sanitation facility of the Household d) An FIR filed against some unknown person. Which of the above indicators is/are covered under the Multidimensional Poverty Index recently seen in the news? Ans) b a) 1, 2 and 4 only Exp) FIR’s are recorded in a register, specifically d) 3 and 4 only maintained for that purpose, in every police c) 1, 2 and 3 only station and given a number. However, if the d) All of the above information is regarding some crime committed at a place that is not covered within the Ans) d jurisdiction of that particular police station, then Exp) All options are correct. that information is recorded, but not numbered, thus it is referred to as ‘Zero FIR’.

The recently published Global Multidimensional Poverty Index (MPI) 2020 by the United Nations Development Programme and Oxford Poverty and Human Development Initiative has jeopardised the advancement in the reduction of MPI due to the impact of COVID-19 on deprivation parameters. The MPI report has cited two areas that have severely affected the deprivation indicators. These are nutrition and children’s school attendance. An analysis of 70 countries shows that the pandemic has disrupted school education globally due to national and local lockdowns, where 91% of the children are out of their Source) schools. https://www.epw.in/journal/2020/34/letters/m The MPI report that has combined the impact of pi-and-covid-19.html attendance (50%) and nutrition (25%) reveals that http://hdr.undp.org/en/2020- the aggregate global MPI of 70 countries can MPI#:~:text=The%20global%20Multidimens change from 0.095 to 0.156 in 2020. This is ional%20Poverty%20Index%20(MPI)%20m almost equivalent to the value of the year 2011 easures%20the%20complexities%20of,povert y%2C%20covering%205%20billion%20peo MPI for India is calculated as 0.123. About 27.9% ple. of the total population includes the multidimensionally poor. The percentage of Q.91) The Metonic Cycle recently seen in the people susceptible to multidimensional poverty is news is related to a 19.3%. The share of deprivation dimension to a) tsunami triggered through periodic total multidimensional poverty in terms of health, earthquakes on the ocean floor. education and standard of living is 31.9%, 23.4% b) periodic change in the sun magnetic field and 44.8%, respecti•vely. However, these figures every 11 year. need to be revised in the context of COVID-19, c) phenomenon of a 19-year cycle related to the and a revised index has to be worked out in the solstice solar eclipse. days to come. d) d)biogeochemical cycle by which organic carbon compounds are exchanged among the biosphere.

Ans) c Exp) Recently, the annular solar eclipse and summer solstice occurred on the same day for the first time in 19 years. Why do we see a solstice solar eclipse in only a gap of 19 years?

The reason for the discrepancy is the cyclical Source)https://www.businessinsider.in/science/s nature of the universe. The phenomenon of a 19- pace/news/the-ring-of-fire-solar-eclipse-and- year eclipse cycle is called the ‘Metonic cycle’. summer-solstice-are-on-the-same-day-for-the- For every 19 years that go by, almost 235 lunar first-time-19-years-and-theyre-only-going-to- months come to pass as well. The 235 lunar get-rarer months fall short by 72 minutes. When they add https://www.timeanddate.com/eclipse/annular- up, they push the Earth out of sync with the solar-eclipse.html solstice and the cycle needs to start over. We just https://www.britannica.com/science/summer- happen to be living at a time which is smack in solstice-astronomy the middle of a lunar cycle that coincides with our annual calendar. Q.92) With reference to Pradhan Mantri Kisan SAMPADA Yojana(PMKSY), consider the An annular solar eclipse happens when the following statements: Moon covers the Sun's center, leaving the Sun's 1. The scheme will provide a big boost to the visible outer edges to form a “ring of fire” or growth of the food processing sector in the annulus around the Moon. During one of the country. phases of the ASE a phenomena called Bailey’s 2. It is a central sector scheme. Beads’ are visible. This is a thin fragmented ring 3. This scheme will submerge all agriculture caused by passage of sunlight through the rough schemes related to farmers. edge of the Moon. During one of the phases of the ASE a phenomena called Bailey’s Beads’ are Which of the statements given above is/are visible. This is a thin fragmented ring caused by correct? passage of sunlight through the rough edge of the a) 1 and 3 only Moon. b) 2 and 3 only Annular solar eclipses can only take place when: c) 1 only ● It is New Moon. d) 1 and 2 only ● At the same time, the Moon is at (or very near) a lunar node, so the Earth, the Moon, Ans) d and the Sun are aligned in a straight (or nearly Exp) Statement 1 and 2 are correct. straight) line. The Central Sector Scheme - SAMPADA ● The Moon is near its farthest point from (Scheme for Agro-Marine Processing and Earth, called apogee, so the outer edge of the Development of Agro-Processing Clusters) was Sun remains visible as a ring of sunlight. approved by the cabinet in May 2017 for the period of 2016-20 coterminous with the 14th Finance Commission cycle. The scheme has now been renamed as the Pradhan Mantri Kisan Sampada Yojana (PMKSY). Summer solstice, the two moments during the Impact of PMKSY year when the path of the Sun in the sky is farthest ● The implementation of PMKSY will result in north in the Northern Hemisphere (June 20 or 21) creation of modern infrastructure with efficient supply chain management from or farthest south in the Southern Hemisphere farm gate to retail outlet. (December 21 or 22).

● It will provide a big boost to the growth of the 2. It is an initiative of the Ministry of External food processing sector in the country. Affairs. ● It will help in providing better prices to Which of the statements given above is/are farmers and is a big step towards doubling correct? farmers' income. a) 1 only ● It will create huge employment opportunities b) 2 only especially in the rural areas. c) Both 1 and 2 ● It will also help in reducing wastage of d) Neither 1 nor 2 agricultural produce, increasing the processing level, availability of safe and Ans) a convenient processed foods at affordable Exp) Statement 1 is correct. prices to consumers and enhancing the export Project Mausam aims to rebuild maritime and of the processed foods. economic connections with the 39 countries bordering the Indian Ocean. The project also aims Statement 3 is incorrect. These schemes are to bring together historical and archaeological under PMKSY(Not all scheme related to researchers to record the diversity of economic, Farmer) cultural, and religious interactions between • Mega Food Parks countries in. There are two visions for Project • Integrated Cold Chain, Value Addition and Mausam: the first is to understand national Preservation Infrastructure cultures between countries and the second is to rebuild communications between countries. • Creation/Expansion of Food Processing/Preservation Capacities About the Project • Infrastructure for Agro Processing Clusters Focusing on monsoon patterns, cultural routes • Scheme for Creation of Backward and and maritime landscapes, Project ‘Mausam’ is Forward Linkages examining key processes and phenomena that • Food Safety & Quality Assurance link different parts of the Indian Ocean littoral as Infrastructure well as those that connect the coastal centres to • Human Resources and Institutions their hinterlands. Broadly, Project ‘Mausam’ aims to understand how the knowledge and Source) manipulation of the monsoon winds has shaped https://vikaspedia.in/agriculture/policies-and- interactions across the Indian Ocean and led to schemes/crops-related/pradhan-mantri-kisan- the spread of shared knowledge systems, sampada-yojana traditions, technologies and ideas along maritime https://mofpi.nic.in/sites/default/files/important_ routes. These exchanges were facilitated by notice-sampada-19.05.2017.pdf different coastal centres and their surrounding

environs in their respective chronological and Q.93) With reference to Project Mausam spatial contexts, and simultaneously had an effect sometimes seen in the news, consider the on them. following statements: The Project scope falls under several themes to be 1. The project aims to rebuild maritime and explored through various UNESCO Culture economic connections with the countries Conventions to which the Government of India is bordering the Indian Ocean.

a signatory with the Ministry of Culture and ASI The scientific objectives of AstroSat mission as nodal agency. are: ● To understand high energy processes in Statement 2 is incorrect. Project ‘Mausam’ is a binary star systems containing neutron stars Ministry of Culture project to be implemented and black holes. by Indira Gandhi National Centre for the Arts ● Estimate magnetic fields of neutron stars. (IGNCA), New Delhi as the nodal coordinating ● Study star birth regions and high energy agency with support of Archeological Survey of processes in star systems lying beyond our India and National Museum as associate bodies. galaxy. ● Detect new briefly bright X-ray sources in the Source) sky. https://thewire.in/diplomacy/mausam- ● Perform a limited deep field survey of the initiative-indian-ocean-world Universe in the Ultraviolet region. http://www.indiaculture.nic.in/project- mausam Source) http://www.newsonair.com/News?title=Indian Q.94) Which of the following is/are objective of -astronomers-discover-one-of-the-farthest- the ISRO Astrosat mission recently seen in the Star-galaxies-in-universe&id=398796 news? https://www.isro.gov.in/astrosat-0 1. To understand the high energy processes in black holes. Q.95) Which of the following statements best 2. To detect new briefly bright X-ray sources in describe the term “Novichok”, sometimes seen in the sky. the news? 3. To estimate magnetic fields of neutron stars. a) A highly toxic nerve agent which can lead to Select the correct answer using the code given death by asphyxiation. below: b) A new Russian Intercontinental Ballistic a) 1 and 3 only missile capable of carrying nuclear warheads. b) 2 and 3 only c) A heat resistant synthetic composite fibre c) 1 and 2 only used in spacecrafts. d) 1, 2 and 3 d) A preservative used in processed and canned foods. Ans) d Exp) All statements are correct. Ans) a AstroSat is the first dedicated Indian astronomy Exp) mission aimed at studying celestial sources in X- Novichok is a highly toxic nerve agent that ray, optical and UV spectral bands slows the heart, paralyses the muscles used for simultaneously. The payloads cover the energy breathing and — if the dose is big enough — bands of Ultraviolet (Near and Far), limited can lead to death by asphyxiation. optical and X-ray regime (0.3 keV to 100keV). A smaller dose may result in seizures, One of the unique features of AstroSat mission is neuromuscular weakness, liver failure and other that it enables the simultaneous multi-wavelength damage. observations of various astronomical objects with More generally, nerve agents are highly toxic a single satellite. chemicals that can either be a gas, aerosol or

liquid, which poison the nervous system and nayaka (chieftain). under the ruler Deva Raya II disrupt bodily functions. also known as Prauda Deva Raya of the Nerve agents can be inhaled, absorbed through Vijayanagara Empire. the skin or ingested through poisoned food or drinks and — depending on the purity — can appear clear and colourless, and can either be odourless or have a faint sweet smell. While we don't know the exact chemical structure of Novichok, it is believed to be five to 10 times more lethal than other nerve agents such as Sarin gas or VX.

Source) ImageSource: https://www.thehindu.com/news/international/ru http://blankslatechronicles.com/virupaksha- ssian-opposition-leader-alexei-navalny- temple-hampi-india-story-mathematical-wonder/ poisoned-by-nerve-agent-novichok- germany/article32507037.ece Option 2 is incorrect. Arjuna’s Penance is one https://www.abc.net.au/news/2020-09-03/what- of the magnificent monuments of is-novichok-russia-alexei-navalny-poison- Mahabalipuram. This wonderful base air relief symptoms-explained/12624142 dates back to the mid-seventh century. Standing

tall at a height of 43 feet, the monolith was carved Q.96) Consider the following on the face of two huge adjoining boulders, architecture/monuments: making its length around 96 feet. 1. Virupaksha Temple 2. Arjun Penance 3. Lotus Mahal Which of the above is/are part of the Group of Monuments at Hampi, a UNESCO World Heritage Site? a) 1 and 2 only b) 2 and 3 only c) 1 and 3 only d) 1, 2 and 3

Ans) c Exp) Options 1 and 3 are correct. Virupaksha Temple is located in Hampi in the Ballari district of Karnataka, India. It is part of the Imagesource: Group of Monuments at Hampi, designated as a https://en.wikipedia.org/wiki/Descent_of_the_G UNESCO World Heritage Site. The temple is anges_(Mahabalipuram)#/media/File:Descent_o dedicated to Lord Virupaksha, a form of Shiva. f_the_Ganges_01.jpg The temple was built by Lakkan Dandesha, a

Option 3 is correct. Lotus Mahal is one of the Q.97) Consider the following pairs: fine architecturally designed palaces that are Terms sometimes Context/Topic uniquely identified by its lotus look alike seen in news structure. This glorious building is within the 1. Wolbachia - A nerve agent Zenana Enclosure, a segregated area that is used 2. Sputnik V -Floating nuclear reactor by the royal women of Vijayanagara Dynasty. 3. Flavonoids - Phytonutrients The balcony and the passages are covered with a Which of the pairs given above is/are correctly dome that looks like an opened lotus bud. The matched? central dome is also carved as a lotus bud. The a) 1 only curves of the palace are given an Islamic touch b) 1 and 2 only while the multi-layered roof design is moreover c) 3 only related to Indo style of buildings. The style and d) 2 and 3 only designs is an inquisitive blend of Islamic and Indian way of architecture. Ans) c Exp) Wolbachia is a genus of intracellular bacteria that infects mainly arthropod species, including a high proportion of insects, and also some nematodes. It is one of the most common parasitic microbes and is possibly the most common reproductive parasite in the biosphere. (Pair 1 is incorrectly matched.)

In 2008, the Australian-based research group World Mosquito Program (WMP) discovered that Aedes aegypti mosquitoes can no longer spread dengue when they are carrying Image source: Wolbachia, This is because the dengue virus https://www.karnataka.com/hampi/lotus-mahal/ struggles to replicate inside the mosquito when these bacteria are present. Source) https://timesofindia.indiatimes.com/city/hubballi Recently Russia registered the world’s first /tourism-activity-picks-up-in- COVID-19 vaccine “Sputnik V”, it was hampi/articleshow/77897376.cms promising news for many who were desperate for https://www.tourism-of-india.com/arjunas- a way to prevent a COVID-19 infection. penance-mahabalipuram.html http://blankslatechronicles.com/virupaksha- Sputnik V uses viral vectors. Viral vectors are temple-hampi-india-story-mathematical-wonder/ genetically engineered organisms that carry a https://en.wikipedia.org/wiki/Descent_of_the_G gene of interest into the human body. For Sputnik anges_(Mahabalipuram) V, an adenovirus acts as the vehicle delivering a gene belonging to the SARS-CoV-2 virus. The vector does not integrate itself into the body’s cells. It simply releases an actual gene of interest

into the body to trigger an immune Q.98) “This Empire arose from the ruins of the response.(Pair 2 is incorrectly matched.) Kampili kingdom in the first half of the 14th century. It grew into one of the famed Hindu Flavonoids are a diverse group of empires of South India that ruled for over 200 phytonutrients (plant chemicals) found in years. It flourished as it controlled cotton and almost all fruits and vegetables. Along with spice trade routes of Southern India. However, carotenoids, they are responsible for the vivid the glory of this empire was short lived as it was colors in fruits and vegetables. Flavonoids are the destroyed in 1565 in the Battle of Talikota.” largest group of phytonutrients, with more than Which one of the following Empires is mentioned 6,000 types. Some of the best-known flavonoids above? are quercetin and kaempferol. (Pair 3 is a) Maratha Empire correctly matched.) b) Bahamani Empire c) Rashtrakuta Empire In recent years, scientists have turned to various d) Vijayanagara Empire flavonoids to explain some of the health benefits associated with diets rich in fruits and vegetables, Ans) d according to the Linus Pauling Institute. Like Exp) In 1336 CE, the Vijayanagara Empire other phytonutrients, flavonoids are powerful arose from the ruins of the Kampili kingdom. antioxidants with anti-inflammatory and immune It grew into one of the famed Hindu empires of system benefits. Diets rich in flavonoid- South India that ruled for over 200 years. The containing foods are sometimes associated with Vijayanagara rulers fostered developments in cancer, neurodegenerative and cardiovascular intellectual pursuits and the arts, maintained a disease prevention. However, it is not yet clear strong military and fought many wars with whether the flavonoids themselves are sultanates to its north and east. They invested in responsible. roads, waterworks, agriculture, religious buildings and public infrastructure. The site used Source) to be multi-religious and multi-ethnic; it included https://indianexpress.com/article/explained/mosq Hindu and Jain monuments next to each other. uitoes-dengue-indonesia-experiment-6572699/ The buildings predominantly followed South & https://www.bbc.com/news/world-europe- Indian Hindu arts and architecture dating to 43377698 & the Aihole-Pattadakal styles, but the Hampi https://www.livescience.com/52524- builders also used elements of Indo-Islamic flavonoids.html & architecture in the Lotus Mahal, the public bath https://indianexpress.com/article/lifestyle/health/ and the elephant stables. do-you-love-jamun-heres-why-you-must-love- its-seeds-too-6483220/ Vijayanagara Empire flourished as it https://science.thewire.in/health/covid-19- controlled cotton and spice trade routes of sputnik-v-vaccine-russia-rdif-adenovirus-vector- Southern India. Medieval historians refer to phase-3/ Hampi as an important centre of trade. However, the glory of Vijayanagara was short-lived. With the death of Krishnadevaraya, the combined armies of the muslim kingdoms- Bidar,

Golconda, Ahmednagar and Berar destroyed this mighty empire in 1565 in Battle of Talikota. Temple architecture in the south attained its climax under the Cholas. The style of Hence, option d is correct. architecture which came into vogue during this period is called Dravida, because it was confined Source) largely to south India. An early example of the https://pib.gov.in/PressReleaseIframePage.aspx? Dravidian style of temple architecture is the PRID=1649971 & eighth century temple of Kailasanath at https://www.britannica.com/event/Battle-of- Kanchipuram. One of the finest and most Talikota & https://frontline.thehindu.com/arts- elaborate examples of the style is, however, and-culture/heritage/article25880107.ece provided by the Brihadisvara temple at Tanjore built by Rajaraja Chola. This is also Q.99) With reference to the Chola Empire of called the Rajaraja temple because the Cholas Medieval India, which of the following were in the habit of installing images of kings statements is/are correct? and queens in the courtyards of the temples. 1. The founder of Chola empire Vijayalaya was The temple at Gangaikondacholapuram, though a contemporary of Harshavardhan. in a dilapidated condition, is another fine example 2. Chola Kings installed images of kings and of temple architecture under the Cholas. Hence, queens in the courtyards of the temples. statements 2 and 3 are correct. 3. The Brihadisvara temple at Tanjore was built by Rajaraja Chola. Source) Select the correct answer using the code given https://www.thehindu.com/news/national/andhra below. -pradesh/rare-inscription- a) 1 and 2 only unearthed/article32490807.ece#:~:text=A%20rar b) 3 only e%20inscription%20dating%20back,fraternity% c) 2 and 3 only 20of%20archaeology%20and%20history. & d) 1, 2 and 3 History of Medieval India – Satish Chandra, Chapter – 3 ‘South India: The Chola Empire Ans) c (900-1200) ’. Exp) THE RISE OF THE CHOLA EMPIRE The founder of the Chola empire was Vijayalaya, Q.100) With reference to the differences between who was at first a feudatory of the Pallavas. He Gross Value Added (GVA) and Gross Domestic captured Tanjore in AD 850. And by the end of Product (GDP), consider the following the ninth century, the Cholas had defeated both statements: the Pallavas of Kanchi (Tondaimandalam) and 1. GVA gives a picture of the state of economic weakened the Pandyas, bringing the southern activity from the producers’ side whereas the Tamil country under their control. Harsh GDP gives the picture from the consumers’ Varshan ruled over Kannauj from AD 606- side. 647. Hence, statement 1 is incorrect. 2. Although measured differently, both are always the same. The greatest Chola rulers were Rajaraja (985- Which of the statements given above is/are 1014) and his son Rajendra I (1014-1044). correct?

a) 1 only b) 2 only c) Both 1 and 2 d) Neither 1 nor 2

Ans) a Exp) What is gross value added? Put simply, it is a measure of total output and income in the economy. It provides the rupee value for the amount of goods and services produced in an economy after deducting the cost of inputs and raw materials that have gone into the production of those goods and services. It also gives a sector-specific picture like what is the growth in an area, industry or sector of an economy.

How is it measured? At the macro level, from a national accounting Source) perspective, it is the sum of a country’s GDP https://indianexpress.com/article/business/econo and net of subsidies and taxes in the economy. my/economy-slowdown-gdp-contraction-minus- When measured from the production side, it is a 23-9-per-cent-6578116/ & balancing item of the national accounts. https://economictimes.indiatimes.com/markets/st ocks/news/the-debate-over-the-use-of-gva-and- What is GDP? gdp/articleshow/58905721.cms & It gives the economic output from the consumers’ https://www.thehindu.com/business/Economy/th side. It is the sum of private consumption, gross e-hindu-explains-what-is-gross-value-added- investment in the economy, government and-how-is-it-relevant-when-growth-is- investment, government spending and net foreign announced-in-gdp-terms/article31768578.ece trade (difference between exports and imports).

What is the difference between the two? While GVA gives a picture of the state of economic activity from the producers’ side or supply side, the GDP gives the picture from the consumers’ side or demand perspective. Both measures need not match because of the difference in treatment of net taxes. This is one of the reasons that in the first quarter of 2015, GDP growth was stronger at 7.5%, while GVA growth was 6.1%. Hence, statement 1 is correct but statement 2 is incorrect.